0% found this document useful (0 votes)
1K views

Gray File HAAD

The document contains a series of multiple choice questions related to pharmacology. It tests knowledge of appropriate drug dosages, drug interactions, contraindications, and diagnoses based on presented medications. The questions cover topics like asthma, COPD, diabetes, gout, hypertension and more.

Uploaded by

ali khan
Copyright
© © All Rights Reserved
We take content rights seriously. If you suspect this is your content, claim it here.
Available Formats
Download as PDF, TXT or read online on Scribd
0% found this document useful (0 votes)
1K views

Gray File HAAD

The document contains a series of multiple choice questions related to pharmacology. It tests knowledge of appropriate drug dosages, drug interactions, contraindications, and diagnoses based on presented medications. The questions cover topics like asthma, COPD, diabetes, gout, hypertension and more.

Uploaded by

ali khan
Copyright
© © All Rights Reserved
We take content rights seriously. If you suspect this is your content, claim it here.
Available Formats
Download as PDF, TXT or read online on Scribd
You are on page 1/ 67

H

H
A
A
A
A
.Gray File
D
D

.Gray File
Dr Akhtar Hussain
HAAD/DOH: Qualified

Conquer the HAAD UAE Exam with Confidence!


Master the essential knowledge with our comprehensive guide featuring a
bank of MCQs meticulously compiled from past exams.

Best Of Luck

Dr Aamir Iqbal 1 Dr Akhtar Hussain


HAAD Test; 15/02/2024
Which drug is suitable for MDS (Monitored Dosage system)?

A) Ibuprofen B) Candesartan C) Methotrexate D) Nicorandil

1) How many tablets you will dispense to pt, presents a prescription with prednisolone. It is advised to
start the therapy with 40 mg for 7 days followed by a reduction of 5 mg every 3 days until finished. How
many 5 mg prednisolone tablets are needed to be dispensed?

A) 140 B) 60 C) 130 D) 160

2) What is the recommended Digoxin dose for a 36 month old infant weighing 17kg?
Age Microgram/kg/day
0-12 months 14
12-24 months 11
2-5 years 9
6-10 years 7
Cal= 17x9=153
A) 153 B) 160 C) 170 D) 180

3) A 78-year-old woman with a history of atrial fibrillation and heart failure is on amiodarone, ACEI and B-
blocker. She has been taking digoxin to improve symptoms and exercise intolerance. She has a normal
renal function. What would be the most appropriate dose of digoxin per day?

A) 62.5 mcg B) 250 mcg C) 125 mcg D) 31.25 mcg

4) A 21-year-old boy came with complaints of shortness of breath he is on Salbutamol inhaler since last 10
years, X-ray show no consolidation, the patient becomes stable on the medication but his sleep breaking
3-4 times night time what advice will doctor give to the pharmacist.

A) Change to LABA. B) Add ipratropium C) Add prednisolone D) Add Montelukast

5) A black race patient is on Amlodipine 5mg his BP was found 164/110mmhg the next adding drug?

A) Candesartan B) Ramipril C) Spironolactone

6) Pt. taking 500mg twice daily tablet wants to shift to oral solution. Pharmacy has 125mg/5ml of oral
solution in stock. The bioavailability of tablet is 0.6 and oral solution is 0.8. How many ml of oral solution
must be given if the prescription calls for 3 days

A) 90ml B) 300ml C) 100ml D) 45ml

7) A patient with asthma was brought to the hospital. She was already taking SABA and ICS for
maintenance. However, she feels scared that another attack might cause her medications to not be
effective any more. What drug is the LEAST helpful should she experience another attack?

A) Theophylline B) Ipratropium C) Prednisolone

Dr Aamir Iqbal 2 Dr Akhtar Hussain


8) What drug is the most helpful if she experience another attack?
A) Theophylline B) Ipratropium C) Prednisolone

9) Patient with COPD is taking tiotropium, the pharmacist know that he should add another drug, what he
should choose?

A) Theophylline B) Salbutamol C) Prednisolone

10) Which of the following is the most suitable method of drug delivery for infants?

A) Neutralizers B) Turbohalers C) Inhalers D) Evohaler

11) In COPD patients what medication should ipratropium be administered with?

A) Albuterol B) Tiotropium C) Prednisolone D) None of the above

12) Patient is taking b-agonist and ICS for COPD and he had oral thrush. What is the proper technique to
avoid it:

A) wash and gargle with water after use


B) Oral thrush is normal while using Corticosteriods

13) Patient is taking b-agonist and ICS for COPD and he had oral thrush. What is the proper technique to
avoid it:
A) use spacer
B) wash mouth with water after each inhalation
C) Oral thrush is normal while using Corticosteriods

14) By using inhaled corticosteroids & short acting beta agonist experience oral thrush. Why?

A) Due to inhaled corticosteroids B) Due to inhaled SABA

15) A patient came with a prescription of salbutamol, Prednisolone, beclomethasone, amoxiclav . What is
the diagnosis?

A) COPD exacerbation B) Asthma exacerbation C) Pleural effusion D) chest infection

16) patient is taking Prednisolone, Amoxicillin and Theophylline. What is the diagnosis?

A) Chest infection w/o asthma and COPD B) Pleural effusion


B) Asthma exacerbation D) COPD exacerbation

17) COPD with penicillin allergy & doctor prescribed an antibiotic with Prednisolone. Which is the antibiotic
choice?

A) Doxycycline B) Amoxicillin C) Metronidazole D) TPM/ SMX

Dr Aamir Iqbal 3 Dr Akhtar Hussain


18) A 72-year-old man with a history of hypertension and COPD has EKG finding suggestive of irregular
rhythm (see vitals). Blood pressure 135/80 mmHg Heart rate 80/min What is the most appropriate drug to
manage his atrial fibrillation?

A) Verapamil B) Digoxin C) Metoprolol D) Warfarin (Target INR of 2-3)

19) Narcotic 1 full stock was missing, was reported to pharmacist in charge then during enquiry was found
that the stock was transferred to another private hospital and the case was noted in the register the next
day. What is wrong in this scenario?

A) Transfer of narcotics to private hospital B) The case was noted next day

20) drug has oral bioavailability of 8 % . one patient takes this drug with grape fruit and found that
bioavailability reduced by 40 % . what is the amount of drug bioavailable if oral dose was 320 mg ?

0.08*320=25.6 ,
25.6*0.06=15.3 mg
According to this formula 15.3mg ans

Amount Bioavailable=Oral Dose×(Bioavailability/100)


According to this formula 217.6mg ans

21) Pt. has UTI with a GFR of 15ml/min. Which drug is contraindicated?

A) Nitrofurantoin B) SMX-TMP C) Co-amoxiclav

22) Patient with infection has penicillin allergy. What is alternative drug?

A) Levofloxacin B) Cefalexin C) Co-fluampicil D) Co-amoxiclav

23) A Diabetic patient came with complaints of swollen foot wound with redness and hot. he is a farmer
working in the field, he is allergic to penicillin antibiotic, and his renal clearance is 70ml/hr.

A) Refer him to the Emergency department or Assist him to the acute care immediately without giving any
pain reliever
B) Give him antibiotic and analgesics
C) Advise him to do dressing for two days if not healing consult doctor

24) A Diabetic patient came with complaints of swollen foot wound with redness and hot. he is a farmer
working in the field, and his renal clearance is 70ml/hr. The doctor ruled out it was a mild infection. What is
the suitable antibiotic- no penicillin allergy.

A) Co-amoxiclav B) Amoxicillin C) Flucloxacillin D) Metronidazole

Flucloxacillin is also use but in this case co amoxiclave is prepared, as co amoxiclave is used for broad
spectrm anti bacterial activety , and the patient is expose to envormental pathognes

Dr Aamir Iqbal 4 Dr Akhtar Hussain


25) A Diabetic patient came with complaints of swollen foot wound with redness and hot. he is a farmer
working in the field, he is allergic to penicillin antibiotic, and his renal clearance is 70ml/hr. The doctor
ruled out it was a mild infection. What is the suitable antibiotic- penicillin allergy.

A) Clindamycin (Dicloxacillin, Cephalexin) B) Co-amoxclave C) cephalosporin

26) Patient come to you with hypoglycemia. When you see the prescription, it was written Insulin GI 10.0
IU. What is the least reason to cause his condition?

A) Insulin was not stored in the refrigerator was stored in room temperature.
B) Decimal point and zero
C) Brand name written in abbreviation.
D) U not unit

27) Patient come to you with hypoglycemia. When you see the prescription, it was written Insulin GI 10.0 U.
What is the (most) reason to cause his condition?

A) Insulin was not stored in the refrigerator was stored in room temperature.
B) Decimal point and zero
C) Brand name written in abbreviation.
D) U not unit

28) patient came to pharmacy for insulin, while talking with her she revealed that she used to store Insulin
in freezer, how would you counsel the patient regarding the storage of insulin in freezer .

A. The freezer temperature will lead to the precipitation of insulin


B. Insulin can be stored in freezer
C. The freezer temperature will affect stability of insulin by decreasing the hydrogen bond that lead to lose
of its 3D structure
D. The freezer temperature will affect stability of insulin by increasing the hydrogen bond that lead to lose
of its 3D structure

29) A lady biker is diabetic, she used to travel by carrying insulin in her bike by using ice pack, once she
complained about precipitation of Insulin what would be the cause.

A) Ice packs were in direct contact with medicine


B) The temperature was not maintained properly

30) A patient with joint pain of metacarpal joints...etc. Renal clearance is around 25 ml/hr, diagnosed with
acute gout. The best drug for treatment is?

A) Ibuprofen B) Allopurinol C) Prednisolone D) Colchicine


Management of Gout in CKD: For acute gout attack, NSAIDs are relatively contraindicated. Lower doses of
colchicine are recommended in CKD but Glucocorticoids ( prednisone, prednisolone, triamcinolone,
methylprednisolone, dexamethasone ) are the preferred agents in renal patients.
In chronic gout, the goal is to maintain uric acid levels < 6 mg/dl.

Dr Aamir Iqbal 5 Dr Akhtar Hussain


31) What happens when insulin gets accidentally frozen?

Precipitation happens B) Efficacy of insulin decreases.

32) A 60 year-old man with a history of heart failure presents with an acute attack of gout in the left toe
with excruciating pain. He rates his pain in his toe as 8/10. Which is the best treatment option?

A) Aspirin 300mg daily B) Diclofenac 75mg daily C)Allopurinol 300mg daily D)Colchicine 600mcg daily

33) Patient taking linagliptin, amlodipine, ibuprofen, ramipril. His creatinine clearance become low. What
medication you think he should stop, if any:

A) Linagliptin, ramipril, ibuprofen B) Amlodipine, linagliptin C) Ramipril, ibuprofen

34) Patient went to pharmacy complaining of severe muscle pain. She said that she was prescribed to take
lansoprazole 2 weeks for GERD. The pharmacist inspected her previously dispensed medications and saw
Salbutamol and Simvastatin. What caused this symptom to appear to the patient?

A) Adverse effect of Simvastatin


B) Drug interaction between her medications
C) Adverse effect of Lansoprazole
D) An underlying disease that was not treated

35) Which PPI is safe with clopidogrel? OR The least interaction producing PPI?

A. Pantoprazole B. Omeprazole C. Ranitidine D. All of the above

36) Patient taking topical corticosteroid nasal spray. What condition is not Contraindicated? Or
contraindicated except?

A) Deviated septum B) Pulmonary TB C) Active infection D) Post-surgery (nose surgery)

37) What is the BEST counseling for a patient applying topical tacrolimus for his psoriasis?

A) Wash hands after use B) Use a sunscreen of at least 15 spf after applying

38) When refill of prescription can be done in the following condition

A) Chronic disease patient with complete management


B) A patient came earlier because she is going for 21 days travel
C) A chronic medicines prescription of 5th refill for 3 months

39) Wrong statement regarding narcotic prescription:

A) Schedule II medications can be refilled ( Codine, morphine, methadone)


B) New prescription must be written every time
C) Medications classified as a schedule III and IV controlled substances may be refilled
D) All of the above

Dr Aamir Iqbal 6 Dr Akhtar Hussain


40) A 20-year-old woman with a history of periodic acute migraine attacks associated with aura is having
another acute attack. What would be the first-choice drug?

A) Sumatriptan 50 mg at the onset of migraine B) Paracetamol 1000 mg four times daily


B) Codeine 30 mg four times daily D) Ibuprofen 400 mg four times daily

41) Sumatriptan is known to be effective for migraine. Which of these situations are contraindicated from
taking it?

A) Pt. with a cardiovascular disease.


B) Pt. who is taking 10 cigarettes per day.

42) A prescription with Amoxicillin 5ml OD, the best way to write the direction?

A) 5 ml 1 teaspoonful 1 time daily


B) 5ml of ONE teaspoonful ONE time daily
C) Five ml of 1 teaspoonful one time daily
D) 1 teaspoonful one time daily

43) Which of the following medications can be used for narcotic addiction?

A) Methadone B) Codeine C) Morphine D)Hydrocodone

44) patient with opioid addiction. He have QT prolongation. What is the appropriate drug?

A) Naloxone B) Methadone C) Buprenorphine D) flumazeni

45) Pt. came to the pharmacy complaining about dizziness, resting tremors and difficulty in stepping the
leg. She also has QT prolongation. These symptoms appears one week ago when the patient take the
restless leg syndrome medication. The pharmacy inspected the bottle of medicine the patient brought and
saw that the label indicated Quinine sulfate but the appearance of the tablets didn’t match the medicine.
What had contributed to this adverse effect

A) Quetiapine 200mg B) Quinapril 40mg C) Quinidine Sulphate 40mg D Quinin 100mg

46) Technician dispensed a drug instead of another. What is the least appropriate method to make sure it
won't happen again:

A) Put it in shelf with Highlight the use


B) Install barcode system
C) Set an alarm to start when dispense
D) LASA drugs Store in separate area

47) Look-alike sound-alike (LASA) medications what measures can be taken to minimize errors

A) Do not purchase similar packing items


B) Store LASA medications nearby
C) Ask the physician not to prescribe the medicine

Dr Aamir Iqbal 7 Dr Akhtar Hussain


48) The medicine which can’t be manipulated by bare hands as that cause skin irritation

A) Chlorpromazine B) Ibuprofen C) Paracetamol D) Cetirizine

49) Precaution while handling Methotrexate tablet

A) Use Hand glove B) Use of N-95 Mask C) Wear Apron D) Wear Goggle

50) Drug asked to put out of its container & put it in a tray which is the drug shouldn't be used to put out of
its original container?

A) Carbamazepine B) Chlorpromazine C) Both A&B

51) Which of the following is a rare side effect of amiodarone (less than 1%)):

A) Corneal micro-disposition B) Peripheral edema C) Hypothyroidism

52) Which of the following is a rare side effect of amiodarone

A) Peripheral edema B) Hyperthyroidism C) Hypothyroidism

53) Phototoxic drug?

A) Amiodarone B) Quinine D) Ibuprofen D) All of the above

54) In Parkinson’s Disease which antiemetic can be used?

A. Metoclopramide B. Domperidone C. Prochlorperazine D. Promethazine

55) In cancer which antiemetic can be used?

A) Metoclopramide B. Domperidone C. Prochlorperazine D. Promethazine

56) A patient is taking Levodopa. Which vitamin is the cause of concern for pharmacists?

A) Vitamin C B) Vitamin B6 C) Vitamin B12

57) A 65-year-old man diagnosed with Type II diabetes five years ago and is on an oral biguanide. His
routine follows up and laboratory work up is scheduled. Which of the following additional tests is
recommended?

A. Vitamin C B. Vitamin D C. Vitamin B12 D. Electrolytes

58) Phenytoin prepared with: NaCl (Normal Saline)

59) Phenytoin precipitate with : Dextrose

Dr Aamir Iqbal 8 Dr Akhtar Hussain


60) A Parkinsonism patient have surgery . He have to go nil by mouth for 2 days. what treatment have to
be followed ?( he taking medicine 200mg 3 times a day before surgery)

A) Omit the dose for 2 days and reduce dose in to two times
B) Change cobeneldopa to levodopa equivalent dose and advice equivalent roticotine patch / Rotigotine
C) Change cobeneldopa to benserazide equivalent dose and advice equivalent lidocaine patch
D) I don’t remember the option it include wording like crushes and based on dose and half life

61) Pharmacist preparing IV preparation by mixing Cyclizine, Dexamethasone and NS 0.9% by using aseptic
technique, after few minutes the mixture become cloudy. The PH of each preparations are 3, 5.5 and 7
respectively, what would be the cause of precipitation?

A) Use of Normal saline


B) The mixture put in low temperature
C) Use of Aseptic technique
D) Cyclizine

62) Pt. with CrCl of 15ml/min and is diagnose with DVT. What is the recommended dose for enoxaparin
Pt weight is 60kg.

A) 60mg OD B) 60mg BID C) 45mg BID D) 45mg OD

63) A 21-year-old boy came with complaints of shortness of breath he is on Salbutamol inhaler since last 10
years, X-ray show no consolidation, now which drug can be added for managing this condition.

A) Beclomethasone B) Ipratropium C) Prednisolone D) Ig antibody

64) The result of one patient revealed hyperkalemia and might escalate to arrhythmia. Her current
medications are ff… What predisposed her to this complication?

Eplerenone and Ramipril

65) Pt. taking 500mg twice daily tablet wants to shift to oral solution. Pharmacy has 125mg/5ml of oral
solution in stock. The bioavailability of tablet is 0.6 and oral solution is 0.8. How many ml of oral solution
must be given if the prescription calls for 3 days.

A) 90 ml B) 300ml C) 100ml D) 45ml

66) A 20-year-old male was admitted to ER because of Asthma exacerbation. He has been taking
salbutamol for 10 years. He has no consolidation. What is the unrelated drug to give?

A) Clarithromycin B) Ipratropium C) Theophylline D) Prednisolone

67) A ward nurse returned Bisoprolol tablet after 11? Days to the pharmacy. She said that the medicine
was stored in a room temp. What will you do?
A .Discard the medicine
B. Put it back to the shelf with the other meds
C. Don’t take it back since it’s been more than 3days for the allowed return

Dr Aamir Iqbal 9 Dr Akhtar Hussain


68) Expires ampoules of morphine. What is the most appropriate action ?

A) Discard in the sharp object box


B) Store in separate place till the next inspection in 10 months
C) Contact Regulatory to make proper disposal and reissue

69) Disposal of medicines in Home?

A) Dissolve in water B) Dissolve in coffee C) Dispose in Chemical bin D) Dispose in Dust bin

70) What is an example of allergy and not side effect?

A. Eczema B. vasculitis C. liver dysfunction D. nephritis

71) Case of patient with mental illness. Caretaker told the pharmacist the patient is not taking the medicine
and his condition is getting worse (violent). What is the best step to take?

A) Plan a meeting among doctor, pharmacist, patients relatives and caretaker to decide what to do
B) The responsible person should evaluate the person's condition
C) Patient has he right to decide whether to take or not to take medicines( if his condition is ok )
D) Drug should not be crushed to mix in food as it will reduce the drugs potency and bioavailability

HAAD Test: 22/01/2024


72) a patient is prescribed prednisolone 40mg for 7 days, followed by a reduction of dose of 5mg for the
next 3 days and continue as upto until it is finished. How many 5 mg tablets have to be dispensed?

A) 130 B)140 C) 100 D) 120

73) Afro patient taking Amlodipine 5mg. His BP is 159/100. Which is the best medicine to be added?

A) Ramipril B) Lisinopril C) Candesartan

74) Patient is taking methotrexate and is known to be excreted 90% unchanged in the urine. Methotrexate
levels diminish at 96 hours. What might be the cause of prolonged excretion of this drug?

A) Concomitant use with sulfamethoxazole.


B) CYP450 inhibition with clarithromycin
C) Increased fluid in body due to effusion

75) A long case of a patient taking Lithium 300 mg TID. His plasma concentration of lithium is 0.7mcg/ml..
What will be the effect of loop diuretics on concentration of Lithium?
A)0.5 B) 0.7 C) 0.9 D) 1.2

76) A long case of a patient taking Lithium 300 mg TID. His plasma concentration of lithium is 0.7mcg/ml..
What will be the effect of thaizide diuretics on concentration of Lithium?
A)0.5 B) 0.7 C) 0.9 D) 1.2

Dr Aamir Iqbal 10 Dr Akhtar Hussain


77) a patient taking Lithium 300 mg TID. His plasma concentration of lithium is 0.7mcg/ml. His plasma
levels decreased to 0.5mcg/ml. What is the appropriate dose adjustment?

A) Lithium carbonate 400mg TID


B) Lithium carbonate 300mg QID
C) Lithium carbonate 400mg QID
D) No adjustment Required

78) Sterilization of patch which is stable at 90 degree celsius.


A) Autoclave B) Ethylene gas C) Uv D) Dry heat

79) Case about a patient with opioid addiction. He have QT prolongation. What is the appropriate drug?

A) Naloxone B) Methadone C) Buprenorphine D) flumazenil

80) Second step in root cause analysis?

A) Provide recommendation
B) Document and research
C) Make action
D) Identify root cause

81) Pharmacist opened a bottle of the drug four days ago. Now some quantity from that bottle needs to be
dispensed to a patient. Expiry date is not mentioned. But he is informed that it will be valid after opening
for 28 days. According to pharmaceutical law it will expire after 6 months, what will be the expiry of the
drug ?

A) 24 days B) 28 days C) 30 days D)60 days

82) A female patient whom you have known for years came to the pharmacy with some allergy after taking
peanuts food from outside. She has a history of frequent anaphylaxis. She forgot her epinephrine injection
and epinephrine was out of stock in the pharmacy. The patient collapsed. What Is the appropriate action?

A) Call the emergency medical team and start cardiopulmonary resuscitation


B) Give her salbutamol for her breathing
C) Raise her leg for 1 minute and wait and don't do anything
D) Give her drug X and wait

83) Least suitable route for peptide administration?


A) IM B) Oral C) SC D) IV

84) A patient came to the pharmacy asking for diazepam. The pharmacist is not in the pharmacy. The
patient claimed that she used to get diazepam from the pharmacist without prescription. What is the
appropriate action you should take?
A) Give her the medicine
B) Refuse to give and document it as error and offer OTC
C) Refuse to give her and offer OTC medicine
D) Refuse to give, offer OTC, document the error, and call the regulations and police

Dr Aamir Iqbal 11 Dr Akhtar Hussain


85) A long case history about patient having COPD. Which of the following is not effective in the acute
management of this patient.

A) Oral prednisolone 50mg for 5 days


B) Nebulized ipratropium 5mg every 6 hrs
C) Nebulized salbutamal 5mg every 6 hrs.
D) Oral azithromycin 500 mg twice a day for 7 days

86) A new pharmacist receive medicine of narcotics, while looking the narcotics register the book was
finished. So he have to write in new narcotics register book . He already included name, strength, quantity
and voucher no . What should be added from the following

A) market authorisation letter B) voucher date D) Medicine source

87) What is the possible serious side effect that is common amongst all anticoagulant agents?

A) Major Bleeding B) Hypokalemia C) Renal Dysfunction D) Liver Dysfunction 23)

88) Long case of Afro-Caribbean patient. He had severe ankle swelling. Which of the following drug cause
this condition

A. Nifedipine b. Losartan c. Metoprolol d. spiranolacone

89) which condition nasal spray of steroid is not contraindicated?

a post nasal surgery b. rhinitis c. nasal infection

90) Patient taking nitroglycerin mononitrate 60 mg XR once daily. He was under control until he started to
have symptoms. He decided to take another dose. What is your appropriate action:

a. Ask physician to change to dinitrate since it has longer duration of action.


b. Ask physician to change to patch to wear throughout the day
c. Tell patient to continue taking the drug twice a day
d. Tell patient to take the drug daily as it was originally prescribed

91) A patient has allergic reaction. He has very short bile. The physician prescribed prednisolone enteric
coated tablet. What is your advice to the physician:

a. Change to prednisolone normal tablet


b. Administer prednisolone enteric coated tablet
c. Change to prednisolone hypo-osmolar solution
d. Change to prednisolone hyper-osmolar solution

92) A patient presents to the pharmacy complaining that few days after starting benzoyl peroxide cream
and soap the acne got worse. What should be the pharmacist advice?
A. Continue the soap
B. Discontinue the soap
C. Continue the cream and soap
D. Discontinue the cream and soap

Dr Aamir Iqbal 12 Dr Akhtar Hussain


93) A patient asks the pharmacist to verify bottle of his medicine. The label was attached to the wrong
medicine and dispensed to the patient by another pharmacist. What should be the most appropriate
response?

A. Apologize and give the correct medication


B. Apologize and give the correct medication and report error to the ministry of the health
C. Apologize and give the correct medication and report error to the medication safety officer
D. Apologize and give the correct medication and ask to report error to the hospital administration

94) A 59-year-old woman with a history of bronchial asthma has been diagnosed with open angle
glaucoma. The doctor is thinking about prescribing a topical eye drop. What would be the best topical eye
drops?

A. Timolol B. Pilocarpine C. Brimonidine D. Latanoprost

95) Which one of the calcium channel blockers is non-dihydropyridine drug?

A. Nicardipine B. Amlodipine C. Nifedipine D. Diltiazem

96) Which of the following is an important counseling point for patients starting on warfarin?

A) Keep intake of green leafy vegetables.


B) Avoid green leafy vegetables.
C) Green leafy vegetables should be taken at least 4 hours after warfarin.
D) Green leafy vegetables should be taken at least 2 hours after warfarin.

97) Phototoxic drug? (It is in long case with a patient have some disease .and taking class III potassium
channel blockers.. and because of that drug he need to use sunscreen like that..) Amiodarone

98) Which drug causes recent fall ? ramipril

99) What is the LEAST appropriate counseling for pt. with eczema?

Using lot of soap and water when taking a bath

100) For a multiple dose vial, what might be the changes in its expiry, just in case?

A) Follow the manufacturer’s expiry


B) No change in expiry
C) Change the shorter expiry date vials to the longer expiry date vials
D) Follow the manufacturer’s expiry OR change to 1 year after you dispensed

101) The patient is taking a monoclonal antibody and she knows how to administer it herself. However, she
noticed a red bump on her arm due to repeated injections on that site. What condition of the skin might be
appropriate to inject the medicine?

A) Apply on sweaty B) Apply on red C) Apply on tender D) Apply on hard

Dr Aamir Iqbal 13 Dr Akhtar Hussain


102) Patient is using benzyl peroxide soup and cream for acne treatment but her acne got worse than
before, what to do:

a) add doxycycline b) stop using immediately c) continue using d) add isotretinoin capsule

103) for management of intra cranial damage patient administered with IV Mannitol, the ward nurse noted
the precipitation of the substances inside the infusion bag. The proper management is /Mannitol 20% is
known to crystallize at higher temperatures. What should be the appropriate precaution before
administering

A) Use filter at the starting of IV fluid line


B) Don’t use Precipitated mannitol bottle

104) A 21 year lady was on Carbamazepine and Phenytoin, but recurrent episode happened so she
changed the medicine to Valproate, then she doesn’t have any seizures for the last five years, her husband
come to pharmacy for medicine and he said she try to conceive. What advice will give you regarding
medication?

A) Consult a neurologist before conceiving and take opinion about continuation of Valproate
B) Continue valproate because it doesn’t cause any teratogenic effect
C) Stop valproate before 3 months of conceiving
D) Change the medicine to another antiepileptic.

105) Pt taking parkinsons disease medication, have symptoms dizziness, fatigue, heavy breathing his hb is 7
what to give him ?

a) Hydroxocobalamin iv every 1-3 monthis


b) Folic acid oral daily
c) Darbepoetin s.c every 3 weeks

106) Why ophthalmic suspension should not be freeze? It will make crystallization

107) Patient is allergic to preservatives what base is suitable to use to make sublingual preparation:

a. Anhydrous base b. Hydrous and oily base c. Water base

108) Patient with breast cancer and hypertension is taking these medications: Tamoxifen, Capecitabine,
Ibuprofen and Lisinopril. She/he developed nephrotoxicity as a result. Which drug should NOT be stopped?

A) Tamoxifen B)Capecitabine C) Lisinopril D) Ibuprofen

109) A Diabetic patient came with complaints of swollen foot wound with redness and hot. he is a farmer
working in the field, he is allergic to penicillin antibiotic, and his renal clearance is 70ml/hr. he wants to
continue working what will be the best advice to him

a) Refer him to the Emergency department or Assist him to the acute care immediately without giving any
pain reliever
b) Give him antibiotic and analgesics
c) Advise him to do dressing for two days if not healing consult doctor

Dr Aamir Iqbal 14 Dr Akhtar Hussain


110) The culture report ;
Cotrimoxazole Sensitive
Clindamycin Resistant
Doxycycline Sensitive
Metronidazole Resistant
Which of the following action is preferable?

a) Administer Doxycycline 200mg, then Doxycycline 100 mg on daily basis for 5days
b) Complete the current course antibiotic for 7 days then start Doxycycline
c) Discontinue the antibiotic and start one of sensitive antibiotic
d) Start sensitive antibiotic along with Current antibiotic and complete the course of 7 days

111) 85 year-old male with chronic kidney disease is diagnosed with UTI. Which drug should not be given?

A. Nitrofurantoin B. Amoxicillin C. Azithromycin D.Ceftriaxon

112) 85yr Patient with infection has penicillin allergy. What is alternative drug?

A) Levofloxacin B) Cefalexin C) Co-fluampicil D) Co-amoxiclav

113) A radioactive substance with radio activity 4380 curium received by pharmacist at Monday 08:00am
for thyroid patient treatment. The optimum radioactivity for thyroid gland treatment is below 230 curium,
if the half-life of substance is 6 hours in what time it can be used. (answers are in 24 hour format )

A) Tuesday 08 am b) Thursday 08 am C) Following week 08am D) Tuesday 02 pm

114) An antibiotic has an elimination rate constant of 0.05 per hour and apparent volume of distribution of
0.6 L/Kg. What will be the clearance value in a 75 Kg adult?

A) 2.25 L/hrs. B) 1 .75 L/hrs. C) 3.55 L/hrs D) 4.25 L/hrs

115) Drug have oral bioavailability 8% and he take it with grape fruit juice so reduce by 40% what is the
amount of bioavailability if the oral dose is 350 mg?
Bioavailability=8%
So the bioavailability of drug is in normal saline solution 320*8/100= 25.6mg
25.6*60/100= 15.36

116) Vial contains 4200 units in 50 ml Dose prescribed 200 units You will dilute the vial with normal saline
0.9% How many ml you will need from normal saline without exceeding the concentration 15 units per ml
After dilution we need 15units/ml
15ml = 1ml
4200units = Xml
Cross multiplication
Xml = 4200x1/15 = 280
As 4200 units already diluted in 50 ml of normal saline

So 280-50 = 230
A) 230ml b) 250ml c) 300ml

Dr Aamir Iqbal 15 Dr Akhtar Hussain


117) Which of the information is found in a morphine Rx but NOT on an amoxicillin Rx?

A) Prescriber’s registration number B) Prescriber’s address C) Prescriber’s telephone

118) While you are writing an information about Oxycodone in Register Book and made a mistake. what
you will do?

A) Write error on the remarks and write on the next line


B) Cross out/make a line on the error and write the correct info beside it
C) Use a correction pen/ink to erase the error
D) Throw away the register and write it in a new one

119) In the noon shift, the in-charge noted the discrepancy in the narcotic stock, next day morning she
informed to the pharmacist and he reported the incident then he asked the incident about responsible
person. He said the Narcotic medicine containing box transferred to another health care facility. In this
incidence, which step deviate from the standard management of Narcotics?

A) The time of reporting incident


B) All comply the standards of narcotic management
C) Transfer of Narcotic to another healthcare facility

120) Best way of prescribing Digoxin 0.25 mg to avoid error ?

A) 250 mcg B) 250 µg C) 250,000 ng D) 250 ug

121) A female reach to hospital for her seizure treatment and after consultation Technician dispensed
chlorpromazine instead of other drug. And the patient started side effects.and reach hospital while
checking prescription and medicine dispensd .it is found to be wrong medicine is dispensed.. what is the
condition occur?

it is due to LASA error instead of carbamazepine technician dispense chlorpromazine

122) Side effect of Ibuprofen?

A) Heart burn B) Edema C) Liver damage

123) At what time should the routine plasma samples for digoxin monitoring be drawn?

A) 6 hours post-dose B) 2 hours post-dose C) 4 hours post-dose

124) What is a rare type of adverse drug reaction?

hyperthyroidism from Amiodarone

125) In What situation should a generic name be okay to use?

A) Amiodarone and the brand is effective for the pt.


B) Valproate and the pt experience adverse effect to that brand
C) Warfarin and it’s the first time to prescribe.
D) Theophylline and the brand is an unsuccessful therapy
Dr Aamir Iqbal 16 Dr Akhtar Hussain
126) ALKALI parenteral solution is to be prepared in an aseptic technique. What type of glass is to be used?

A. Type I B. Type II C. Type III

127) An intravenous preparation of Methotrexate is prepared which is the best glass material for this
preparation?

a. Type I glass b. Type II glass c. Type III glass d. PVC bottle

128) Non aqueous preparation & powder for PWT use -

A) Type III glass B) Type ll C) Type l

129) Aqueous preparation of powder for Perentals use. Which type of glass?-

A) Type III glass B) Type ll C) Type l

130) A Y drug is administered which is sensitive to plastic material and moisture. What is the most
appropriate action ?

a) Exposure to UV light before use b) Stored in type-I glass container

131) A 68-year-old woman with type 2 diabetes, neuropathy, and seizures has been newly diagnosed with
depressive illness. Which medication would be best to initiate?

A. Bupropion B. Duloxetine C. Paroxetine D. Amitriptyline

132) Disposal of medicines by pharmacist

a) Dissoleve in water b) Dissolve in cofee c) Dispose in Chemical bin d) Dispose in Dust bin

133) Phenoarbital preparing solution

a) DW5% b) Water for injection c) Normal saline

134) A patient is taking Mebaverine, Methotrexate and Azathioprine what would be the disease?

a) Crohns Disease b) Rheumatoid Disease c) Osteomyelitis

135) What should not be manipulated by bare hands due to its effect?

A. Cetirizine B. Ibuprofen C. Paracetamol D. Chlorpromazine

136) 440mg of Gentamycin wa given as IV infusion to a patient yesterday. The nurse checked the drug level
and it was 2 mg/ml today. The normal level og Gentamycin is < 1mg//ml. what will you do?

a) Advice to check the drug level again


b) Tell the doctor to check the levels after 6 hrs and give same dose until it reaches < 1 mg/ml
c) Call and confirm the time of administration to the nurse

Dr Aamir Iqbal 17 Dr Akhtar Hussain


137) After consultation Pharmacist needs to update a patient’s record. Which should not be longer
included?

A. Allergy B. Cost of medicine C. Indication

138) What is NOT the appropriate measure when cleaning a vertical laminal flow hood?

A. Use of isopropyl alcohol and leaving it for 30 secs


B. Disinfecting the area BEFORE and AFTER every batch
C. Food and beverage can be stored in ante area and not in the buffer room
D. D. Disinfecting the instrument

139) Pt. went to pharmacy complaining of severe muscle pain. She said that she was prescribed to take
lansoprazole 2 weeks for GERD. The pharmacist inspected her previous dispensed medications and saw
Salbutamol and Simvastatin. What caused this symptom to appear to the pt?

A. Drug interaction between her medications


B. Adverse effect of Simvastatin
C. Adverse effect of Lansoprazole
D. An underlying disease that was not treated

140) What is the recommended Digoxin dose for a 36 month old infant weighing 17kg?
Age Microgram/kg/day
0-12 months 14
12-24 months 11
2-5 years 9
6-10 years 7
Age*Microgram/kg/day = 9*17= 153

Ans: 153

141) What is NOT true about these statements regarding hydrocodone?

A. a licensed physician can prescribe up to 3 days maximum only


B. the prescription validity is only 3 days starting from the day it was prescribed
C. narcotic prescriptions should be kept in the pharmacy for 5 years
D. there is no refill for hydrocodone

142) A company is marketing a new drug and these are the specifications
Stomach - absorbed
t 1/2 - 2 hrs.
Packaged in a 1,000 Units in a box
The use of coating is justified for what reason?

a. To handle the attrition and abrasion


b. To prevent the active ingredients from being exposed to oxygen
c. To flow smoothly on the machine
d. To prevent the drug from degrading

Dr Aamir Iqbal 18 Dr Akhtar Hussain


143) 21 year lady was on Carbamazepine, but no use. Took sodium Valproate for 5years without any
seizure attack for 5years ,She has come to pharmacy with husband to take valproate tabs and have plans
to start a family .

A. Stop sodium valproate and wait for 14days washout period then plan
B. Can continue as no danger on pregnancy
C. Continue and then discontinue when become pregnant
D. Consult a neurologist before conceiving and take opinion about continuation of Valproate

144) Prescription contains simvastatin, and insulin lispro, which one should be dispensed in brand name

A. Simvastatin B. Insulin lispro C. Both D. None of those

145) Prescriber now prescribed deferoxamine, rifampicin Earlier the drug in the prescription was ascorbic
acid and dapson. What is the patient disease condition?

A. Hemochromatosis and TB B. Leprosy and scurvy


B. TB and Leprosy D. Hemochromatosis and Leprosy

146) patient history carbidopa, levodopa……which antiemetic should give for controlling vomiting in this
patient

A. Metoclopramide B. Domperidone C. Prochlorperazine

147) Sterilization of highly reactive material which is moisture sensitive which method used for sterilization

A. Depyrogenation B. Filtration C. Chemical substance

148) Pfizer covid vaccine was opened yesterday afternoon but the nurse for got to keep it in the fridge.
What can we do

A. Can be used today morning B. Discard as no efficacy C. Can be used till tomorrow evening

149) Side effects of b Agonist?

A. Palpitation B. Sweating C. Extrapyramidal side effect D. Cough and wheezing

150) A physian prescribed Depakote chrono against valpro chrono is available?

A. It can be dispensed because both have same pharmaceutical active ingredient


B. It can’t be dispensed because this drug have narrow therapeutic range
C. It can’t be dispensed because both of them have different Active pharmaceutical ingredient
D. The prescribed drug should be dispensed because of dr specifically mentioned the brand name.

151) Pt feeling lightheadness which medicine she used?

A) Atenolol and lisinopril


B) Atorvastatin and lisinopril
C) Frusemide and Calcium gluconate

Dr Aamir Iqbal 19 Dr Akhtar Hussain


152) Dose of aspirin?

A. 75mg once B. 300 mg twice C. 65 mg once

153) calaculation questions of flurouracil? Ans: 18.3 see question 499 for calculation

154) overage calaculation? Ans) 6.94 see question 500 for calculation

155) calculation etoposide question? Ans 5 ml see question no 501 for calculation

HAAD Test : Dec 2023

156) A drug was changed from schedule V to III on April 1st. How you will register it?

A) Continue registering it in the same register


B) Continue as up to and after April 1st registering for schedule V & schedule III respectively
C) Register as schedule V until April 1st then registers as schedule III in their register separately

157) Which drugs are put in the pharmacy separately along with their register:

a. Narcotics, psychotropics, semi-controlled


b. Narcotics, psychotropics ( controlled ), semi-controlled, unlicensed
c. Narcotics, psychotropics( Controlled )

158) Preregistered pharmacist and locum pharmacist got close. The preregistered pharmacist noticed that
the locum pharmacist is ill & started to go to the bathroom more frequently. What he shall do?

a. Call the regular pharmacist and ask for advice


b. Talk to him privately in the counselling room in lunch break
c. Talk with his colleagues to see what to do with him
d. Don’t bother him

159) Digoxin dose for 57 kg adult with 15mcg/kg calculate the maximum daily dose

A) 855 mcg B) 865 mcg C) 854 mcg

160) A prescription came to you by patient having a certain drug which is not in your stock and the stock is
already empty for that specific drug , what will you do ?

a) Ask the patient to go to another pharmacy


b) Give him another drug from the same class

161) Which is least important in prescription?

a) Name and sign of the Pharamacist c) Name and sign of the prescriber
b) Name and emblem of Pharmacy d) Name and sign of technician

Dr Aamir Iqbal 20 Dr Akhtar Hussain


162) You got prescription stating to prescribe Ramipril 100mg but the previous history of patient showing
that patient having 10mg. you are noticed prescription error. You are calling GP but he is not attending
phone call, what you will do now?

a) Call the clinic of and ask them to contact GP


b) Don’t prescribe medicines
c) Yourself correct the prescription to 10mg and dispense the medicines

163) Nurse came to the pharmacy. She needed 4 ampoules of pethidine urgently for a patient and there is
a shortage in the country. At the same time another patient returned 8 unused ampoules. What is the least
appropriate reaction:

a. Dispense the medicine for her c. Suggest another pain medication


b. Tell her you don’t have it d. Discuss opioids alternative with the physician

164) Long case of Afro-Caribbean patent taking nifedipine, and other drugs . He had severe ankle swelling.
Which of the following would be temporary fix for his condition?

A. switch to losartan and furosemide


B. Switch to Amlodipine (dose) and Bendroflumethiazimorde

165) Biweekly means:

a. Once every 2 weeks (every other week)


b. Twise weekly

166) Which of the following represent medication error F?

a. Patient take bisoprolol 5 mg instead of biscodyl 5mg , and admitted to the hospital because he
experienced dizziness
b. Patient take 100 mg insulin from expired vial but he experienced no harm.
c. Error that lead to Surgical Debridement

167) Which of the following is an additive drug ?

a) Clonazepam b) Phenytoin c) Fentanyl

168) 28 tablets of morphine were received from company. In the same time a patient come with
prescriptions of 28 tablets of morphine. What is the least appropriate reaction:

a) Dispense morphine after probable documentation


b) Dispense morphine and don’t document it
c) Dispense morphine and document it tomorrow
d) Dispense morphine and after probable documentation contact the company to supply another amount

169) A certain medicine is short in whole UAE, you only have little stock of it. A nurse came to you
prescription of that you have a good terms with that nurse, what will you do?

a) Give her that medicine b) Give her alternative of that medicine

Dr Aamir Iqbal 21 Dr Akhtar Hussain


170) Patient returned 28 tablets of Morphine to pharmacy and on the same day after same time a
prescription came for 28 tablets of Morphine, what will you do?

a) Dispense that 28 tablets to the new patient


b) Don’t dispense him
c) Dispense that 28 tablets with proper inventory documentation

171) A physician prescribed a cream for 1 month yr old baby starting with CLOBETA... REMAINING CANT
READ PROPERLY. The strength of the cream is mentioned as 0.05%,which is these can be consider
regarding the prescription ?

a) The prescription is legally invalid b) Consider it has clobatasone c) Ask the mother

172) Malaria question. A family of parents and 2 children are planning to travel to malaria prone country
for 6 weeks. Physician prescribed chloroquine + proguanil combination tablets for 6 weeks. What is your
advice to the physician: ?

a. Parents should take the prophylaxis


b. All family members should take prophylaxis
c. Only parent should take prophylaxis, children don’t need it

173) Patient with granulating red wound with moderate exudate what is the appropriate dressing?

A) Low adherent dressing a. Iodine b. Honey c. Hydrocolloidal

174) A patient with a prescription of methotrexate and latanoprost come to you. What is the diagnosis: ?

a. Rheumatoid arthritis & glaucoma


b. Osteoarthritis & glaucoma
c. Rheumatoid arthritis & conjunctivitis
d. Osteoarthritis & conjunctivitis

175) Which of the following is an adverse drug reaction:

A. Alendronate acid increase osteonecrosis


B. Albuterol bronchodilation
C. Hypotension of an antihypertensive drug
D. Hypoglycemia due to sulfonyl urea

176) Which of the following is rapid acting insulin (Doctor prescribed 10 IU Three times a day Which of the
following) :?

A. Humalog b. Lantus c. Levemir d. Tresiba

177) morphine and opioids cause which of the following:

A) Increase pain threshold


B) Pinpoint pupils with increased respiratory rate
C) Decrease pain threshold
D) Dilated pupils with increased respiratory rate
Dr Aamir Iqbal 22 Dr Akhtar Hussain
178) Dimethicone mechanism of action:

a. Interfere with water balance


b. Causes paralysis
c. Interfere with metabolism
d. Interfere with O2 demand

179) A 52 female suspected to have osteoporosis. She has hot flushing and undergoes hysterectomy. What
is the most appropriate treatment?

a. unopposed estrogen b. Bisphosphonate c. Estrogen & progesterone.

180) Which of the following is long-acting b-agonist:

a. Formoterol b. Albuterol c. salbutamol

181) Patient taking SABA and ICS and Montelukast( LTRA). He started to suffer from side effects from
montelukast. What is the alternative:

a. oral zileuton b. Inhaled c. theophylline d. Zafirlukast

182) Hygroscopic substance needs to be coated what is the least suitable material to use:

a. Calcium chloride b. Iron oxide

183) Which of the following is enzyme inducer: (all are enzyme inhibitor except)

a. Carbamazepine b. Fluoxetine c. Cimetidine d. Amiodarone

184) Drug cannot be used in MDS:

A. Letrozole b. Atorvastatin

185) You notice ice on the refrigerator, what vaccine will be affected:

a. HPV b. BCG c. MMR d. Varicella zoster

186) A patient came as representative for his father that he has Severe Pneumonia. Physician advised for
new drug inhalation therapy. He gave his father the new medicine by nebulizer. But his father developed
severe bronchoconstriction after using the drug. What counselling will you give to the patient?

A. Colistimethate sodium is produce bronchoconstriction while doing inhalation therapy to avoid this it has
to be mixed with bronchodilator
B. Colistimethate sodium is isotonic with body, it has to be mixed/diluted with normal saline for
nebulization
C. Colistimethate sodium is a potent drug
D. Colistimethate sodium must be administered after taking the solution into body temperature

Dr Aamir Iqbal 23 Dr Akhtar Hussain


187) Pharmacist was covering a shift replacing another. The narcotics were in double lock cabinet in
separate room away from public. Which of the following did not breached protocol?

a) The pharmacist who isn't the owner can dispense narcotics


b) There was no CCTV
c) The cabinet was not fixed to the wall or part of immovable system
d) The register and prescription forms were not inside the cabinet

188) A 16 year-old girl with asthma recently had aroutine laboratory follow up (see lab results potassium
level , low) Which medication is most likely to have contributed to her results?

A) Inhaled short-acting B2 agonist


B) Inhaled corticosteroid
C) Leukotriene receptor antagonist
D) Inhaled short-acting anti-muscarinic

HAAD Test 27/01/2024

189) A drug that cause agranulocytosis ?

a) Captopril( rare ) b) Fenoldopam c) Levoflgrodine

190) Mother came with child having symptoms like nausea, vomiting blood. The test result showed he had
metabolic acidosis. Which of the following is the cause ?

a) Acetaminophen b) Aspirin c) Iron d) Ascorbic acid

191) Father of a 6 years old child came to the pharmacy with prescription of amoxicillin (dose) syrup. So,
what is the best advise as a pharmacist ?

a) Should be tightly cezeded b) Shake well before in each use c) Use 5ml spoon for measurement

192) Patient using hyaluronate ophthalmic drug for 5 months now. Its expiration date is on December 25th.
What you will advise ?

a) Discard since it is opened for 28 days


b) Continue using it didn’t complete 6 months and it’s not expired
c) Discard before 2 days of expiry
d) Discard before 2 days of expiry date and buy new one

193) Narcotic ampoules breakdown. What is not appropriate method of disposal/discard ?

a) Discard in the sharp object box


b) Store in narcotic cabinet
c) A reagent ( can’t remember ) pour to narcotic ampoules
d) Discard with a witness

Dr Aamir Iqbal 24 Dr Akhtar Hussain


194) Trainee pharmacist ordered a huge amount of omeprazole accidently. What is the least appropriate
to do ?

a) Exchange with other pharmacy


b) Ask physician to prescribe it more often
c) Ask patient to tell prescriber to add it to their prescription

195) Why glyceryl trinitrate cannot be stored in PVS container ?

a) Glyceryl will be absorbed in PVC (polyvinyl chloride) and affect its stability
b) The PH of PVC will change through time resulting it ineffectiveness of glyceryl
c) PVS container cause chemical interaction and increase the microbial growth within glyceryl tablets

196) Soft gelatin capsule most appropriate storage ?

a) Glass container with desiccant b) PVC container without desiccant

197) Which is not preferable storage condition for soft gelatin capsules ?

a) Clear glass with desiccant


b) Amber glass without desiccant
c) Polyethylene terephthalate (PET) container
d) High density polythene (HDPE) container

198) A Y drug is administered which is sensitive to plastic material and moisture. What is the most
appropriate action ?

a) Exposure to UV light before use b) Stored in type-I glass container

199) Patient taking metformin 500 twice daily, long acting insulin once daily, short acting insulin 3 times
daily. His blood glucose levels are not under controlled. He did not change his lifestyle or diet. What is your
advice ?

a) Ask prescriber to increase dose of insulin


b) Ask prescriber to increase metformin dose to 3 times daily
c) Ask prescriber to monitor his blood glucose closely after meal and adjust his insulin dose according to it.
d) Change the short acting insulin to long acting insulin

200) You notices ice on the refrigerator and all shelf of refrigerator are filled with vaccine packs and
temperature on the thermometer show 0-2 oC and set off the alarm. What might be the problem ?

a) Thermometer is not working


b) The alarm is set on low degree
c) There is no sufficient coolant inside
d) The ventilation is blocked

201) You noticed ice on refrigerator. What vaccine will be affected ?

a) BCG b) MMR c) Varicella Zoster d) Human Papilloma vaccine

Dr Aamir Iqbal 25 Dr Akhtar Hussain


202) A 75 year old lady with GFR greater than 75 ml/min treatment including drug diazepam, indapamide,
and other drug (can’t remember). Patient experience dizziness, fainting, and changes in pulse or heart rate.
What is not or least appropriate reason for patient fall ?

a) Diazepam cause fall


b) Use of indapamide & diazepam in patient GFR 75ml/min
c) Additive effect due to concomitant use of diazepam Indamine

203) Which of the following is enzyme inducer ?

a) Rifmamficin b) Fluoxetine c) Cimetidine d) Amiodarone

204) Patient was using cream and she had rash (small blisters with yellow exudate) what is the name of
this condition ?

a) Impetigo b) Dermatitis c) Cellulitis d) Eczema

205) Patient on community accrued pneumonia. Patient is sensitive to flucloxacillin. Which if the following
most appropriate drug ?

a) Cephalexin b) Penicillin c) Azithromycin

206) Female on valproate wants to get pregnant. What is your advise ?

a) Ask prescriber to lower the dose of valproate


b) Ask prescriber to add phenytoin
c) Ask prescriber to change to lamotrigine
d) Continue the same regimen

207) Patient having heart burn and she took Gaviscon but it didn’t go away. The pain is radiating to neck
and left shoulder. What you should do ?

a) Refer her to family physician


b) Give Omeprazole
c) Suggest not to eat spicy food

208) Patient having heart burn. After discussing, pharmacist ensured that it’s not serious. It’s happened
after she drank a lot of coffee last night. She tried antacid but she is still in trouble. What is appropriate
action ?

a) After consultation give Gaviscon


b) Refer to her family physician
c) After consultation give Omeprazole

209) A patient was prescribed atorvastatin, amlodipine and other drug. What is the appropriate
counselling ? If you have any uncontrolled muscle pain or myopathy you should report

Dr Aamir Iqbal 26 Dr Akhtar Hussain


210) Patient was admitted to hospital with asthma, arrhythmia or AF & peptic ulcer. He took many
medication (digoxin and other medications). His potassium levels were 2.8 mmole/L. Which drug is
responsible for his abnormality ?

a) Digoxin b) ACE inhibitors c) Ranitidine d) Salbutamol

211) Patient on amiodarone & other drugs. She started to have palpitation, anxiety & weight loss. Which
drug is associated her condition ?

a) Hyperthyroidism caused by amiodarone

212) Patient taking supplements with Levothyroxine and her T4 is very low. What supplement is
responsible to this ?

a) Vitamin C b) Calcium Carbonate c) Vitamin D d) Zinc

Calcium carbonate (Os-Cal, Tums) can reduce thyroxine bioavailability by adsorption in an acid
environment. Th ese should be administered separately to avoid interaction.

213) Waste product what is not correct ?

a) Waste product should be kept in separate away from patient and food
b) Each waste bag should be labelled with names of source and destination
c) Waste room should not be full make room for inspection for leaking.
d) Waste should be disinfected before final disposal

214) A family of parents and 2 children are planning to travel to malaria prone country for 6 weeks.
Physician prescribed chloroquine + Proguanil combination tablets for 6 weeks. What is your advice to the
physician ?

a) All family members should take the prophylaxis for 11 weeks


b) Parents should take prophylaxis for 10 weeks and children for 7 weeks
c) Parents should take prophylaxis for 6 weeks and children for 5 weeks
d) Only parents should take prophylaxis, children don’t need it

215) Female patient previously prescribed Folic acid, Lamotrigine. Now prescribe Isotretinoin for acne.
Which of following is most concern ?
a) Family history b) Pregnancy status

216) Patient prescribed antibiotic, but he has risk for tendon rapture. Which antibiotic he should not take ?

a) Ampicillin b) Flucloxacillin c) Ofloxacin d) Cephalexin

217) Patient come to you with hypoglycemia. His fasting blood glucose level was 3,5 moles/L. He is taking
Insulin. What is the most appropriate advise ?

a) Increase the insulin dose


b) Insulin dose should be adjusted so that the normal glucose level will be b/w 4-7 mmoles/L
c) Insulin dose should be continuous because it is already in the normal glucose level b/w 3-7 mmoles/L

Dr Aamir Iqbal 27 Dr Akhtar Hussain


218) Which of the following insulin is given three times daily ?

a) Lantus b) Levemir c) Tresiba d) Humalog

219) Pharmacist was covering a shift replacing another. The narcotic were in double lock cabinet in
separate room away from public. Which of the following did not breached protocol ?

a) There was no CCTV


b) The cabinet was not fixed to the wall or part of immovable system
c) The register and prescription form were not inside the cabinet
d) The register and prescription form were inside the cabinet

220) Which drugs are put in the pharmacy separately along with their register ?

a) Narcotic, psychotropic, semi controlled and unlicensed


b) Narcotic, psychotropic, semi controlled
c) Narcotic & psychotropic
d) Narcotic

221) Drug x ampoules were found to be leaking. Who is responsible to recall all the drug from the market ?
a) Physician b) Nurse c) Pharmacist d) Manufacturer

222) A patient brought a drug back to pharmacy because it was not effective. After inspection it was found
to be expired. What is the least to be done, if any, to ensure this won’t happen again ?

a) Pharmacist should inspect all medicines in store


b) Pharmacist should inspect all medicines in bulk, shelf and store
c) Pharmacist should inspect the presence of expiry date receiving and dispensing
d) Nothing to be done its manual error can’t be prevented

223) Which of the following monoclonal antibody

a) Imatinib b) Transtuzumab

224) A patient is in some sort of pain and he is allergic to diazepam relieve his pain. His eGFR is
30ml/min/1.73m2 . Which of the following he should take ?

a) Diclofenac b) Citalopram c) Clonazepam d) Amitriptyline

225) Patient with depression and have headache that do go away although he used a lot of analgesics,
codeine, PCM & other analgesic drug. What contributed to his headache ?

a) Depression cause his headache b) Headache is due to misuse of analgesic

226) Hand written prescription with Beclo. which is not correct ?

a) The prescription is legally correct


b) Assuming it be closone & dispense
c) Ask his mother if she know and confirm with doctor

Dr Aamir Iqbal 28 Dr Akhtar Hussain


227) Patient is taking B agonist and ICS for COPD and he had oral thrust. What is the least proper advise ?

a) Use spacer
b) Wash mouth with water after each inhalation
c) Change B agonist to anti muscarinic

228) Shortage of one drug in country and an illegal supplier offer it. What to do ?

a) Don’t respond and tell doctors there is shortage in the drug and offer a substitute
b) Don’t respond & tell doctors, nurse there is shortage in the drug and prescribe a substitute
c) Don’t respond and tell doctors there is a shortage in the drug and wait ministry of health (MOH) to
decide the best substitute

229) Long case of Afro Caribbean patient taking nifedipine and lisinopril, he had severe ankle swelling.
Which of the following would be temporary fix his condition ?

a) Switch Amlodipine & Bendroflumethiazide


b) Decrease the dose of nefedios & furosemide 25mg
c) Continue Lisinopril 40mg and add Carvedilol 25mg
d) Switch to losartan 50mg & Furosemide 20mg
230) Patient come with prescription of methimazole, mebeverine. You noticed on the system that he was
taking carbazole & Loperamide ? What is the condition ?

a) Hyperthyroidism & IBS


b) Hyperparathyroidism & IBS
c) Hyperthyroidism & ulcerative colitis
d) Hyperparathyroidism & ulcerative colitis

231) Technician dispensed diphenhydramine instead of dimenhydrinate. Which is the least appropriate
method to make sure it won’t happen again ?

a) Put it in shelf with lighting (high light source)


b) Install bar code scan
c) Set an alarm to start when dispense LASA drug

232) Patient was taking mesalamine, few days later he developed nausea & vomiting, palpitation &
flushing. After asking he revealed that he drunk 2 cups of wine. The bottle of the medication was inspected
& upon comparison of mesalamine tablets shape you discover it was another drug. What is that drug ?

a) Mesalamine b) Mebeverine c) Metronidazole d) Sulfasalazine

233) Patient with unstable angina. He was taking nitroglycerine & besoprolol but still not controlled. What
is the next medication that should he consider adding ?

a) Nifedipine b) Amlodipine c) Ranolazine

Dr Aamir Iqbal 29 Dr Akhtar Hussain


234) Patient on amiodarone and metformin. He had mild diabetic infection & the physician prescribed
clarithromycin. What you will advise ?

a) There is interaction b/w amiodarone clarithromycin so amiodarone shall be stopped & continue after
antibiotic treatment
b) There is interaction b/w metformin & clarithromycin
c) There is interaction b/w amiodarone & clarithromycin. So, it shall be changed to another antibiotic like
flucloxacillin
d) There is mild interaction b/w clarithromycin & amiodarone. Consider monitoring

235) Women come to you with her daughter that is suffering from headlice. She already had been treated
with insecticide before but it returned. What is the appropriate reason ?

a) Give permethrin b) Give diamethicone c) Give tea tree oil d) Refer to physician

236) Adverse drug reaction should not report to ?

a) Pharmacy manager b) National health authority c) Local health authority

237) Which of the following is an adverse drug reaction ?

a) Albuterol cause bronchodilation


b) Warfarin increase prothrombin time
c) Alendronate acid increase osteonecrosis
d) Antihypertensive drug decrease the blood pressure

238) Amoxicillin prescription with dose of 500mg 3 times daily for 28 days. What is the most appropriate
response ?

a) Dispense it as it is
b) Dispense after indication confirmed
c) Ask physician to change to 500mg 3 times for 7 days
d) Dispense after indication confirmed with the physician

239) A patient come to pharmacy with his inhaler. He wants to know how to use it. You don’t have it in
your pharmacy. What is most appropriate response ?

a) Tell him to go to another pharmacy


b) Tell him to come back tomorrow when you get the inhaler
c) Tell him to come back tomorrow and watch videos meanwhile to learn who to demonstrate its use
d) Open his inhaler & show him how to use.

240) Filgrastim was diluted with 500ml instead of 100ml. cloudy solution appears. What us the cause ?

a) Precipitation b) Contamination

Dr Aamir Iqbal 30 Dr Akhtar Hussain


241) Mother came to pharmacy with her daughter that has rash in her neck and face. Her mother told you
she has measles. The child used to have fever in the past few days, but she seems fine now. What is the
best treatment ?

a) Acyclovir b) Ibuprofen c) Calamine lotion

242) Patient come to dispense clotrimazole. You noticed that had dispensed it many times in the past few
months. She is diabetic and hypertensive. She is taking gliclazide and amlodipine. What is the most
appropriate action ?

a) Ask physician to change to another antifungal


b) Ask physician to monitor her blood glucose. It may be the cause of her repeated thrasher.
c) Ask physician to monitor her blood pressure. It may be the cause of her repeated thrasher.
d) Don’t changed it is normal side effect with her medication

243) Patient with constipation she noticed blood in her stool. What is the most appropriate treatment ?

a) Fybogel b) Glycerin soap c) Lidocaine Cream

244) Which of the following drug should be administered at the same time daily ?

a) Levodopa b) Atorvastatin c) Levothyroxine

245) Regulatory authority of health changes some guidelines related to pharmacy. Whose responsibility is
to train the pharmacist ?

a) Regulatory of pharmacy
b) DOH authority
c) Pharmacist himself
d) Regulatory of pharmacy responsible for profession

246) Pharmacist woke up not feeling well. What he should do ?

a) Ask the night pharmacist to cover his shift


b) Call the pharmacy manager to arrange replacement.

247) What is the least relevant to drug use ?

a) Patient age b) Patient partner c) Patient pre-medication

248) Which of following is not relevant in prescription bill ?

a) Name and address of prescriber


b) Name and sign of dispensing pharmacist
c) Name and sign of pharmacy technician who compound the medicine

Dr Aamir Iqbal 31 Dr Akhtar Hussain


249) Biweekly prescription for a monoclonal antibody. What to do ?

a) Dispense once every other week


b) Dispense twice weekly
c) Check in dictionary for meaning / check previous record patient Rx
d) Call and confirm to doctor

250) What is not a side effect of contraceptive pills ?

a) Migraine b) Breast cancer c) Ovarian Cancer d) Increased risk of DVT

251) A patient came to pharmacy and he said that he was purchased one medicine 2 weeks ago. His friend
taking the same medicine and he said that this brand has recalled 6 months ago. On verification it is found
that he is correct. Who is responsible for this error ?

a) Patient b) Physician c) Manufacturer d) Pharmacist

252) A patient is taking levothyroxine from last 2 years. She is taking amiodarone from last 2 months, her
thyroid function test in this time found as following
T3 – Very small variation
T4 – within normal limits
TSH – Very high
a) Amiodarone induced hypothyroidism
b) Pituitary adenoma
c) Antibody induced hyperthyroidism
d) Spontaneous hyperthyroidism

253) Administration of BOTOX injection ?

a) IV b) DC c) IM d) Oral

254) 6 years old child with 19kg prescribed digoxin. She need 25mcg/kg once daily. What is the dose of
digoxin in mcg ? Child weight = 19 kg
Dose = 25 mcg/kg
= 19 x 25 = 475 mcg
a) 435 b) 725 c) 475 d) 444
255) A patient weight is 50 kg with a prescription of prednisolone 50 mg for 2 weeks, a dose reduction of 5
mg for one weeks until it is finished. How many pills are needed for 5mg prednisolone ?
50 x 14= 700
45 x 7 = 315
40 x 7 = 280
35 x 7 = 245
30 x 7 = 210
25 x 7 = 175
20 x 7 = 140
15 x 7 = 105
10 x 7 = 70
5 x 7 = 35
700 + 315 + 280 + 245 + 210 + 175 + 140 + 105 + 70 + 35 = 2275
2275 / 5 = 455 pills
a) 475 b) 338 c) 455 d) 454

Dr Aamir Iqbal 32 Dr Akhtar Hussain


256) Insulin degludec pen 200 unit/ml insulin pen contain 3ml. The recommended dose is 25 unit/day.
How many pen needed for 120 days ?
200 unit/ml pen contain = 3ml
3ml contain = 3 x 200 = 600
3ml = 600 units
25 units per day So, 120 days need = 25 x 120 = 3000 units = 3000/600 = 5 pens
a) 3 b) 4 c) 5 d) 6

257) Estriol pump actuation, the dose recommended is 1.5 mg of estradiol for 160 days. One pump delivers
1.25g of 0.75mg of estradiol. The whole pump pack contain 80g. How many pump pack needed for this 160
days ?
dose 1.5mg for 160 days
1.5 x 160 = 240
1 pump delivers 125g of o.75mg drug
Cross multiplication
1.25g = 0.75mg
80g = X mg
X = (80 x 0.75) / 1.25 = 48
For 160 days 240 mg needed 1 pump contain total 48 mg drug So, 240/48 = 5 pump
a) 3 b) 4 c) 5 d) 6

258) Patient was prescribed prednisolone 10 mg for 14 days, then 7.5 mg for 1 week, then 5 mg for 1 week,
then 2.5 mg for 4 weeks. How many tablets of 2.5mg prednisolone you will need ?

10mg - 14 days
10/2.5 = 4 x 14 = 56 pills
7.5mg - 7 days
7.5/2.5 = 3 x 7 = 21 pills
5mg - 7 days
5/2.5 = 2 x 7 = 14 pills
2.5mg - 28 days
2.5/2.5 = 1 x 28 = 28 pills
56 + 21 + 14 + 28 = 119 tablets

a) 140 b) 117 c) 119 d) 256

259) Prescription of 0.3% w/v gentamycin drop. One drop every 2 hours for 5 days then one drop four
times daily for 48 hours in both eyes. What is the amount in mg of gentamycin used in the whole duration ?
0.3 % w/v
0.3 g in 100 ml
300mg in 100 ml
1 drop x 12 x 5 = 60
1 drop x 4 x 3 = 8
68 x 2 = 136 drops
1 ml = 20 drops
X = 136 drops
X = 136/20 = 6.8 ml
300 mg in 100 ml
X mg in 6.8 ml
X = (300x6.8)/100 = 20.4 mg
a) 30 b) 40 c) 20.4

Dr Aamir Iqbal 33 Dr Akhtar Hussain


260) Patient is on Metformin and Amiodarone, Prescribed him Clarithromycin upon calling doctor he
mentioned that clarithromycin prescribed him for mild infection. What will you advice doctor?

a) There is interaction between amiodarone and clarithromycin, amiodarone should be stopped and
continue after resolution.
b) There is interaction between metformin and clarithromycin, metformin should be stopped
c) Clarithromycin should be changed to flucloxacillin
d) There is interaction between Amiodarone and Clarithromycin and consider monitoring

Metformin and Clarithromycin Interaction: Clarithromycin has the potential to increase the levels of
metformin in the body, leading to an elevated risk of lactic acidosis.
Amiodarone and Clarithromycin Interaction: Both amiodarone and clarithromycin can prolong the QT
interval, increasing the risk of ventricular arrhythmias.

261) Controlled drug prescribed for in-patient, how long is prescription validity:

a) 24 hours b) 48 hours c) 72 hours d) 1 week

262) Surgeon woke up in the morning having symmetrical stiffness in both hands.. He has many operations
this week, which the longest duration analgesic he can take:

a) Ibuprofen b) aspirin c) naproxen d) ketorolac

263) Nilotinib is used for :

a) ulcerative colitis
b) Philadelphia chromosome positive chronic myeloid leukemia
c) multiple sclerosis

Nilotinib is a tyrosine kinase inhibitor used in the treatment of Philadelphia chromosome-positive chronic
myeloid leukemia (CML). It works by inhibiting the activity of the BCR-ABL protein, which is characteristic of
CML.

264) Milliseconds receptors and seconds to min receptors ?

a) G protein coupled receptors and voltage gated channels


b) voltage gated channels and G protein coupled receptors
c) voltage gated channels and ligand gated channels
d) ligand gated channels and G protein coupled receptors

Milliseconds receptors typically refer to ligand-gated channels, as the response time is relatively fast,
occurring in milliseconds.Seconds to minutes receptors are more commonly associated with G protein-
coupled receptors (GPCRs). GPCRs transmit signals through intracellular second messenger systems,
leading to cellular responses that often take seconds to minutes.

Dr Aamir Iqbal 34 Dr Akhtar Hussain


265) Chemotherapy patient being treated with capecitabine. 30% decrease in Nutrophil count noted what
should be done?

a) Administer g-csf ( Granulocyte-Colony Stimulating Factor) and cese chemotherapy.


b) Administer g-csf after 72 Hours of Chemotherapy
c) Administer g-csf afer chemotherapy
d) Adminster g-csf after 24 hours of chemotherapy.

266) Which of the following agents is considered a poisonous substance according to HAAD regulations?

A - Chenopodium oil B - Picrotoxin C - Codeine and its salts D - All of the above

Schedule I: High potential for abuse, no accepted medical use, .Examples: Heroin, LSD, MDMA (Ecstasy),
Schedule II:High potential for abuse, accepted medical use, severe physical or psychological dependence may
occur.Examples: Morphine, Oxycodone, Methylphenidate (Ritalin), Fentanyl
Schedule III:Potential for abuse less than Schedule I or II, accepted medical use, moderate to low physical dependence, and
high psychological dependence.Examples: Anabolic steroids, Codeine mixtures, Ketamine
Schedule IV:Low potential for abuse relative to Schedule III, accepted medical use, limited dependence.
Examples: Alprazolam (Xanax), Diazepam (Valium), Lorazepam
Schedule V: Low potential for abuse relative to Schedule IV, accepted medical use, limited dependence.
Examples: Cough preparations with less than 200 milligrams of codeine per 100 milliliters or per 100 grams, (Ezogabine)

267) Which of the following statements regarding hydrocodone is false or not true?

A - A licensed physician can prescribe hydrocodone for a maximum of 3 days only.


B - The prescription validity for hydrocodone is only 3 days, starting from the day it was prescribed.
C - Narcotic prescriptions, including hydrocodone, should be kept in the pharmacy for 5 years.
D - There are no refills allowed for hydrocodone.

268) What is the recommended method for storing a substance containing 70% alcohol?

A - Inflammable substance area


B - Open area
C - Single lock container
D - Double lock container

269) What is the maximum day supply a pharmacist can dispense for a diazepam prescription?

A - 7 days B - 14 days C - 21 days D - 30 days

270) Who bears the responsibility when an infection is attributed to an intravenous (IV) infusion?

A - Pharmacist B - Nurse C - Manufacturer D - All equal

271) Purified water having bacteria of gram +ve and gram -ve, this water is used for manufacturing of the
product, which method is used for the sterilization.

A) Autoclave B) Ethylene gas C) Uv D) Dry heat

Dr Aamir Iqbal 35 Dr Akhtar Hussain


272) Which anticoagulant works by inhibiting adenosine diphosphate receptor:

a) Abciximab b) Acetyl salisylic acid c) Prasugrel d) Unfractionated heparin

273) Drug overdose caused agitation and irritability :

a) Morphine b) Fentanyl c) Cannabinoids

274) Patient took overdose of diazepam and treated with Flumazenil but patient is still hypnotised.. Why?

a) flumazenil is not antidote of diazepam


b) flumazenil dose is not enough
c) flumazenil is not effective after 4 hours of overdose

275) One of these drugs has narrow therapeutic index :

a) Carbamazepine b) Lithium c) ceftriaxone d) iron

276) A 70-year-old woman is diagnosed with sustained ventricular tachycardia. The resident doctor wants to initiate one
of the Class lll potassium channel blockers. She has a history of Torsades de Pointes arrhythmia and has renal impairment.
Which of the following medications is best to initiate?

A) Amiodarone B. Dofetilide C. Ibutilide D. Sotalol

277) Which of the following signs corelate with phenytoin level of 200 micromole/L?

A. Coma B. Ataxia C. Nystagmus D. Decreased mental status.

Less than 10 mg/L: Rare side effects


Ten to 20 mg/L: Occasional mild horizontal nystagmus on lateral gaze
Twenty to 30 mg/L: Nystagmus
Thirty to 40 mg/L: Ataxia, slurred speech, tremors, nausea, and vomiting
Forty to 50 mg/L: Lethargy, confusion, hyperactivity
Greater than 50 mg/L: Coma and seizures

278) What dose high volume of distribution indicate?

A. Drug concentrates in the blood


B. Drug accumulates in the tissues
C. Drug crosses blood brain barrier freely
D. Drug distributes equally between the blood and the tissues

279) A58-year-old woman presents to the Emergency Department with severe right flank pain, nausea and
vomiting. She had urinary frequency and dysuria for the last three days. Her past medical history includes
glucose-6-phosphate dehydrogenase deficiency. What would be the best treatment option?

A. Nitrofurantoin for 5-7 days


B. Cephalexin for 5-7days
C. Ciprofloxacin for 5-7 days
D. Co-trimoxazole for 5-7 days

FQs (Cipro), sulfa drugs (co-trimazole) & Nitrofurantoin are not recommended in G6PD deficiency pts

Dr Aamir Iqbal 36 Dr Akhtar Hussain


280) Which antidepresent drug must be avoided in 21 yr old feamale with a history of seizure?

A) amitriptyline b) bupropion c) Paroxitine D) citalopram

281) A 68-year-old woman with type 2 diabetes, neuropathy, and seizures has been newly diagnosed with
depressive illness. Which medication would be best to initiate?

A. Bupropion B. Duloxetine C. Paroxetine D. Amitriptyline

282) A 10-year-old obese girl with newly diagnosed depressive illness is being advised to start
antidepressant treatment. Which of the following is best to initiate?

A. Bupropion B. Paroxetine C. Mirtazapine D. Amitriptyline

283) A 69-year-old man on routine follow-up is complaining of erectile dysfunction. He has a history of
angina, hypertension, and diabetes mellitus. His medications include daily Aspirin, Metoprolol, Metformin
and Nitrates as required for his angina pain. Which of the following recommendations would be best?

A. Avanafil B. Bupropion C. Sildenafil D. Non-pharmacological treatment

284) Antidepressent used in epilipsy with or without HTN ?

A. Bupropion B. Paroxetine C. Mirtazapine D. Amitriptyline

285) You have a stock of 1%w/v of esterdiol cream . How many mgs you need from this stock to be added
to a cream base to prepare 30 ml of esterdiol cream 3 mg/0.5 ml
3mg in 0.5ml
Xmg in 30ml
Cross multiplication
Xmg= 30x3/0.5=180mg
1% = 1g in 100ml convert 1g into mg = 1000mg
1000mg in 100ml
180mg in Xml cross multiplication
Xml = 180x100/1000 =18ml

A) 14 b) 15 c) 18 d) 12

286) Steve-Johnson syndrome that will appear when giving phenytoin IV after? -

A) 7-14 days after 1st dose -


B) 7-14 days after 2nd dose -
C) 1-2 hours from the 1st dose -
D) 3-5 days after 1st dose

SJS is caused by a drug, symptoms appear about one to three weeks after you start taking medication

Dr Aamir Iqbal 37 Dr Akhtar Hussain


287) Patient taking metformin 500 twice daily, long acting insulin one dose, short acting insulin 3 times
daily. His blood glucose levels are not under controlled. He did not change his lifestyle or diet. What is your
advice:

A) Ask prescriber to monitor his blood glucose closely after meal & adjust his insulin dose according to it
B) Ask prescriber to increase metformin dose to 3 times daily
C) Ask prescriber to increase dose of insulin

288) Patient taking glipizide/glicizide and is having hypoglycemia episodes what to do :

a) switch to another sulfonylurea


b) switch to insulin
c) stop it she don’t need drugs
d) switch metformin to produce normoglycemia

289) A company make a product for colon cancer, its easily absorbed from the intestine , which statement
is not concern with the drug

A) the ph of component 6 to 7
B) the component is gastric acid resistant
C) use of HPMC as coating
D) The drug disrubted in the colon to give action

290) Which of the following medication is most appropriate in a 71-year-old man to treat extrapyramidal
effects emerging out of neuroleptics?

A. Benztropine B. Haloperidol C. Donepezil D. Ibuprofen

291) Which of the following medications require observing the patient for six hours with hourly heart rate
monitoring after the first dose?

Fingolimod A. Verapamil B. Bisoprolol C. Amlodipine

292) An elderly patient is suffering from depression was given St. John's wort. Which of the following drug
if administered concomitantly will have clinically significant interaction?

A. Simvastatin B. Salbutamol C Sertraline D. Gliclazide

293) A 62-year-old man has been admitted in the surgical ward for 14 days and it's suspected to have
methicillin-resistant staphylococcus aureus infection (MRSA) in his wound. He is started on Vancomycin
1500 mg IV as a loading dose followed by 1000 mg every 12 hours. he has normal renal function, and his
weight is 70 kg. What is the recommended time to take Vancomycin serum levels?

A) 1 hr After the first dose B.30 min After the 3rd dose C . After the second dose D. After the 4th dose

294) A 38-year-old man with invasive aspergillosis is receiving voriconazole. Which of the following disease
states warrant dose adjustment?
Hepatic cirrhosis A. Colon cancer B. Renal failure D. No dose adjustments is needed

Dr Aamir Iqbal 38 Dr Akhtar Hussain


295) A woman on mycophenolate is planning for pregnancy. She is asking for pharmacist advice, what
should be the appropriate advice?

A. Mycophenolate should be continued during pregnancy.


B. Mycophenolate should be discontinued at least 6 weeks prior to trying to conceive.
C. Mycophenolate should be continued for the first trimester then discontinued.
D. Mycophenolate should be discontinued at least 12 weeks prior to trying to conceive.

296) patient took iv infusion phenytoin dr changed to tablet 200 mg once daily. The blood level of
phenytoin is 6 the ideal between 10-20. What should the dr do to achieve the optimum level Tablet half-
life is 22 hours

A) Continue same regimen.


B) Give loading dose 400 mg with his same dose this will be in cordarone
C) Change to 200 mg bid
D) Give 400 mg phenytoin once and recheck level after 3 days

297) Case about asthmatic pt. before 2 years, he is taking salbutamol 1- 2 puffs pen, it aggregates with
exercise, he got asthmatic attack & went to hospital, what is the advice?

- Salbutamol 1-2 puffs 10-15 mins before exercise and 1-2 puff in case of symptomatic max 8 puffs a day
- Salbutamol 2-4 puffs 10-15 mins before exercise and 1 puff in case of symptomatic, max 12 puffs a day
- Salbutamol 2-4 puffs 15 mins before exercise and 1-2 puff in case of symptomatic, max 12 puffs a day
- Salbutamol 1-2 puffs 10 mins before exercise and 1 puff in case of symptomatic max 8 puffs a day

298) A 48-year-old man with myocardial infarction is started on an ACEI. He is started on a low dose and
the aim is to titrate to the target dose within four weeks. On routine follow up of labs one week from
starting the drug, his serum creatinine is increased by 20% from the baseline. What would be the most
appropriate action?

A. Stop the ACEI


B. Keep the current dose of ACEI
C. Substitute the ACEI with an ARBs
D. Continue titrating the ACEI

299) An eight-month-old boy with recurrent otitis media has been taking high dose amoxicillin (90 mg/kg)
for seven days. There is no fever and no evidence of hearing loss. He is due his 3rd dose of pneumococcal
vaccination and the influenza vaccination. The nurse is asking about due vaccination. What would be the
best action?

A. Cancel pneumococcal and influenza vaccination.


B. Postpone pneumococcal and influenza vaccination for one year.
C. Delay pneumococcal and influenza vaccination till antibiotic use
D. Proceed with pneumococcal and influenza vaccination

Dr Aamir Iqbal 39 Dr Akhtar Hussain


300) Which diuretic listed below would have the greatest blood pressure lowering effect when given in
patients with an eGFR below 60 ml/min?

A. Chlorthalidone 25 mg daily (thiazide-like diuretic )


B. Spironolactone 25 mg daily
C. Triamterene 25 mg daily
D. Furosemide 40 mg daily

301) A 55-year-old man with myocardial infarction presents to the clinic with dyspnea interrupting his daily
activity. Which of the following medications can cause it?

Ticagrelor A. Aspirin B. Enalapril D. Carvedilol

302) 52 Year type1 diabetes. In the morning feel fatigue and drowsiness . he take insulin before meals and
before bed time . What is the least cause for these symptoms :

A) Fasting blood glucose 5 and blood glucose level 9 before lunch


B) Fasting blood glucose 4 and blood glucose level 7 before lunch
C) Fasting blood glucose 7 and blood glucose level 4 before lunch

303) Which one of the following is considered a dangerous abbreviation?

A) IU for international unit


B) Cap for capsule
C) Q6hr for every six hours
D) IVP for intravenous push

304) The nurse is asking the pharmacist recommendation to reducing the inadvertent intrathecal
administration of vincristine. Which one of the following is recommended?

A) Dispense vincristine in mini bag instead of syringe


B) Dispense the syringe without needle
C) Dispense vincristine in 60 ml syringe
D) Dispense the vial for the nurse to prepare at the bedside

305) patients who require dual antiplatelet therapy, what is the recommended maintenance dose of
aspirin when used in combination with ticagrelor?

A 81 mg B. 162 mg C 325 mg D. 50 mg

306) In a hemodynamically stable patient, when it is recommended to draw gentamycin trough level?

A. At steady state before the fourth dose


B. At steady state after the first dose
C. At steady state after the sixth dose
D. At steady state after the second dose

307) Patient taking gentamicin, which of the following are increased side effects:
Increased thirst must be report to doctor

Dr Aamir Iqbal 40 Dr Akhtar Hussain


308) What is the best time to do peak levels of gentamicin?

A. One hour post-dose


B. Two hours post-dose
C. Three hours post-dose
D. Four hours post-dose

309) What would be the most suitable time to do the peak levels of Vancomycin?

A. Just after the 4th dose


B. 2 hours after the 2nd dose
C. 30 mins after the 3rd dose
D. 1 hour after the 1st dose

310) A 24-year-old man is diagnosed with psychosis. He agrees to start drug therapy, however, he is asking
for the least sedating medication. Which of the following drugs will be most appropriate?

A. Clozapine B. Quetiapine C. Olanzapine D. Risperidone

311) Patient with hypertension was rushed to the ER due to signs and symptoms of acute kidney injury.
These are the drugs she/he were taking: Candesartan, Atorvastatin, Aspirin, Ibuprofen, Bisoprolol,
Paracetamol What drugs should be discontinued? -

A) Stop candesartan, aspirin and ibuprofen -


B) Stop candesartan, bisoprolol and atorvastatin -
C) Stop candesartan, aspirin and paracetamol -
D) Stop candesartan, atorvastatin and paracetamol

312) A 28 year-old man who was well controlled on phenytoin is brought to the Emergency Department in
a state of status epilepticus. What would be the best recommendation?

A. Oral clonazepam and phenytoin


B. Intravenous lorazepam and stop phenytoin
C. Rectal diazepam and intravenous lorazepam
D. Increase dose of phenytoin and give intravenously

313) Which of the following strategy can be recommended to reduce metformin associated
gastrointestinal side effects?

A. Administer 2 hours after meals


B. Administer with H2 receptor antagonist
C. Administer before meals on empty stomach
D. Administer in divided doses with meals

314) Which one of the following medications is considered a high alert drug? Which one of the following
medications is considered a high alert drug?

A. Insulin B. Finasteride C. Ceftriaxone D. Ciprofloxacin

Dr Aamir Iqbal 41 Dr Akhtar Hussain


315) What is the effect of smoking on serum olanzapine levels?

A) Decrease in serum olanzapine levels


B) Increase in serum olanzapine levels
C) Decrease in serum olanzapine levels initially then increase in it
D) Has no effect

316) What is the recommendation regarding air bubble in the syringe for a patient starting on enoxaparin
subcutaneous injection?

A. Inject the dose with the air bubble


B. Inject the air bubble if the volume is small
C. Remove the air bubble to avoid wastage of the dose
D. Remove the air bubble and adjust volume before injection

317) A 72-year-old man with a history of hypertension and COPD has EKG finding suggestive of irregular
rhythm (see vitals). (Blood pressure 135/80 mmHg) ( Heart rate 80/min ) .What is the most appropriate
drug to manage his atrial fibrillation?

A. Digoxin B. Verapamil C. Metoprolol D. Warfarin (Target INR of 2-3)

318) What is the recommendation for administering live vaccine and an antibody containing product?

A. Separate the two administrations by two weeks


B. Administer both at the same visit but different sites.
C. Separate the two administrations by one month and administer at different site
D. Administer the live vaccine first then the antibody containing product one week after

319) A 25 year-old woman with a history of migraine associated with aura has taken tablet ergotamine 2
mg sublingual at first sign of migraine, then 2 mg every 30 minutes as needed. What is the maximum
number of ergotamine tablets she can use in one week?

A. 4 B. 5 tab C. 6 D. 7 Maximum 5 per week and 3 per day

320) A 21 year-old woman with a history of chronic migraine associated with aura is asking for medication
to prevent another attack. What would be the recommended prophylaxis?

A. Sumatriptan 50 mg daily
B. Amitriptyline 50 mg at night
C. Propranolol 20 mg twice daily
D. Topiramate 25 mg twice daily

321) Which one of the following medications is known to induce pulmonary toxicity?

A. Amiodarone B. Metoprolol C. Carbamazepine D. Glibeniclamide

Dr Aamir Iqbal 42 Dr Akhtar Hussain


322) The nurse is asking the pharmacist about recommendations for administering inactivated and live
vaccine. What would be the appropriate recommendation?

A. Both vaccines can be administered simultaneously or at any interval


B. Administer the live vaccine first then the inactivated vaccine one week after
C. Administer the inactivated vaccine first then the live vaccine one week after
D. Separate between the two vaccines by one month and administer at different site.

Both vaccines can be administered simultaneously or at any interval. This choice reflects the Centers for
Disease Control and Prevention (CDC) guidelines, which state that live and inactivated vaccines can indeed
be administered concurrently or at any interval apart if not given on the same day.

323) The nurse is asking the pharmacist about recommendations for administering two and live vaccine.
What would be the appropriate recommendation?

A) Both vaccines can be administered simultaneously or they should be spaced at least 4 weeks apart
B) Administer the live vaccine first then the inactivated vaccine one week after
C) Administer the inactivated vaccine first then the live vaccine one week after
D) Separate between the two vaccines by one month and administer at different site.

324) Which one of the following drugs is considered first line treatment for Alzheimer's disease?

A. Donepezil B. Amantadine C. Pramipexole D. Trihexyphenidyl

325) A 70-year-old woman is taking multiple drugs According to Beers criteria, which one of the following
drugs is potentially inappropriate since it might be associated with increased cognitive decline?

A. Aspirin B. Warfarin C. Enalapril D. Amitriptyline

326) A pediatric patient who was recovering from brain tumor surgery is given 650 mg dose of
Methotrexate into the brain. what kind of error is this?

A. Wrong Drug B. Wrong Dose C. Wrong route D. Wrong time

327) A pregnant lady with body weight of 65 kg, planning to visit a malaria-prone area the very next day.
want to take a daily prophylactic medicine. Suitable prophylaxis is?

A. 325 mg of chloroquine Base


B. Doxycycline 750 mg
C. 250 mg of atovaquone + 100 mg daily
D. 100 mg chloroquine + 200 mg proguanil

328) Glucagon may be used and as an antidote for which of the following?

A. Amitriptyline B. Metoprolol C. Furosemide D. Digoxin

329) Heparin is chemically bonded to catheters to prevent?

A. Thrombogenicity B. Biocompatibility C. Radiopacity D. Bio inertness

Dr Aamir Iqbal 43 Dr Akhtar Hussain


330) A female patient, who has a history of frequent anaphylaxis, came to the pharmacy with food allergy.
she forgot her epinephrine injection; she has difficulty breathing and collapsed what to do?

A. Call the emergency medical team and start cardiopulmonary resuscitation


B. Give her diphenhydramine
C. Give her salbutamol for her breathing
D. Raise her leg for 1 minute and wait and don't do anything

331) A 61-year-old woman with RA experienced Progressive central vision loss and scotoma affecting her
reading ability, A fundus examination showed bilateral bull’s eye maculopathy. Which drug likely to cause
this SE?

A. Hydroxychloroquine B. Prednisolone C. Aspirin D. Adalimumab

332) A 28 year-old man who was well controlled on phenytoin is brought to the Emergency Department in
a state of status epilepticus. What would be the best recommendation?

A. Oral clonazepam and phenytoin


B. Intravenous lorazepam and stop phenytoin
C. Rectal diazepam and intravenous lorazepam
D. Increase dose of phenytoin and give intravenously

Lorazepam, given intravenously, is a first-line treatment for status epilepticus due to its rapid onset of
action and effectiveness in controlling seizures.

333) Patient found tablet on the floor at home, he asked you about it and you know about it, it is for
elderly not for him, what should you do in this situation?

a) Inform him about its use


b) Show him a website about it
c) Don’t inform him since it is not for him and tell him to check with physician.
d) Tell him you don’t know

334) A 79-year-old man with Alzheimer's disease has mild to moderate dementia and is on oral
rivastigmine 6 mg twice daily. He is not able to tolerate the medication due to nausea and vomiting. What
would be the best recommendation?

A) Change to transdermal application of rivastigmine 4.6 mg/ 24-hours


B) Suppositories of prochlorperazine 25 mg 60 mins before each dose of rivastigmine
C) Discontinue rivastigmine and substitute with galantamine 4 mg twice daily
D) Metoclopramide 10 mg 30 mins before each dose of rivastigmine

335) The nurse come to you asking about which type of dressing to be used in a patient having lower leg
wound which is yellow (infection) and perfusing.

A. Low adherent dressing B. Iodine dressing C. Honey dressing D. Colloidal dressing

336) One of these is resistant to penicillinase enzyme:


A) flucloxacillin B) ampicillin C) penicillin g

Dr Aamir Iqbal 44 Dr Akhtar Hussain


337) A 27-year-old pregnant woman has been recently diagnosed with iron deficiency anemia. Her
hemoglobin is less than 11 g/dl. What would be the best recommended?

A. Ferrous sulphate tablets with cup of tea/coffee


B. Ferrous sulphate tablets on an empty stomach
C. Ferrous sulphate tablets with antacids
D. Ferrous sulphate slow-release tablets

338) Which of the following medications require observing the patient for six hours with hourly heart rate
monitoring after the first dose?

A. Verapamil B. Bisoprolol C. Amlodipine D. Fingolimod

339) Which of the following is an important counseling point for woman starting on fingolimod

A. Use effective contraception to avoid pregnancy during and 2 months after discontinuing treatment
B. Pregnancy should be avoided for 6 months after discontinuing treatment
C. Use effective contraception to avoid pregnancy during treatment
D. Fingolimod is safe during pregnancy

340) Which medication requires negative pregnancy test before dispensing to woman of childbearing age?

A. Labetalol B. Prednisone C. Ceftriaxone D. Isotretinoin

341) The physician is asking pharmacist about the effects of low albumin and acute renal failure on
phenytoin serum concentration. The patient is receiving phenytoin 300 mg once daily. What should be the
most appropriate suggestion?

A. In hypoalbuminemia and acute renal failure obtain free phenytoin serum concentration.
B. In hypoalbuminemia and acute renal failure obtain peak phenytoin serum concentration.
C. In hypoalbuminemia and acute renal failure obtain total phenytoin serum concentration.
D. In hypoalbuminemia and acute renal failure obtain trough phenytoin serum concentration.

342) Which of the following is a high intensity statin?

Atorvastatin 40 mg Simvastatin 40 mg Pravastatin 40 mg Lovastatin 40 mg HS

OR set 2 answers:

A. Rosuvastatin 20 mg once daily


B. Atorvastatin 20 mg once daily
C. Simvastatin 20 mg once daily
D. Pravastatin 40 mg once daily

343) A 29-year-old pregnant woman been recently diagnosed with open angle glaucoma. What would be
the treatment of choice?
A. Topical timolol 0.1 gel once daily
B. Topical latanoprost 50 mcg/ml
C. Topical bimatoprost 300 mcg/ml

Dr Aamir Iqbal 45 Dr Akhtar Hussain


344) A 68-year-old woman with a history of type-2 diabetes and dyslipidemia is on atenolol and lisinopril.
The doctor has started simvastatin as her estimated 10-year ASCVD risk is 7.5%. What would be the
recommended dose for simvastatin?

A. 20 mg B 10 mg C. 40 mg D. 80 mg

345) A 55-year-old man is prescribed simvastatin for hyperlipidemia. He is already taking verapamil for
hypertension. What should be the maximum dose of simvastatin to be prescribed?

A. 10mg B. 20mg C. 40mg D. 80mg

346) Which of the following indications is appropriate for misoprostol?

A. Misoprostol is ineffective for prevention or treatment .


B. Misoprostol is effective for both prevention and treatment
C. Misoprostol is effective for preventing NSAIDs-induced ulcer

347) Which of the following pairs is considered as acceptable patient identifiers?

a) Medical record number and full name


b) First name and birth-date
c) Gender and medical record number

348) Why is medication formulated in XR formulation?

A. To easily crush tablet B. To mask the bitter taste C. To release the active ingredient slowly

349) A 75-year-old obese man has been recently diagnosed with osteoarthritis of the knee. He has tried
non- pharmacological measures such as weight loss and various aids to improve symptoms, but he still has
pain in the knee while walking. What would be the best initial treatment?

A. Topical NSAIDs B. Topical glucosamine C. Oral COX-2 inhibitors D. Oral non-selective NSAIDs

350) An 88-year-old frail woman with osteoarthritis has difficulty in walking and pain. She has no other
medical history and has failed numerous aids to help her with the pain. Which of the following drugs is the
best recommendation?

A. Acetaminophen B. Glucosamine D C. Celecoxib D. Ibuprofen

351) A 48-year-old women with rheumatoid arthritis and on ibuprofen and ranitidine for three months has
been advised to start a DMARD ( Disease modifiyng anti rehumatic drug) therapy. Which of the following
would be the recommended DMARD therapy?

A Methotrexate 7.5 mg weekly with folic acid 5 mg weekly on any other day
B Methotrexate 7.5 mg daily with folic acid 5 mg daily
C Methotrexate 7.5 mg daily with folic acid 5 mg monthly day
D Methotrexate 7.5 mg weekly with folic acid 5 mg weekly on the same day

Dr Aamir Iqbal 46 Dr Akhtar Hussain


352) A 52-year-old woman with NYHA Class III Heart Failure was started on lisinopril 40 mg/day and
carvedilol 25 mg BID. She is recently complaining of dizziness. Blood pressure 100/70 mmHg Heart rate
70/min What is the best approach for Heart failure and her symptoms?

A Decrease lisinopril dose B. Decrease carvedilol dose C. Stop lisinopril D. Stop carvedilol

353) A 26-year-old man is coughing up greenish yellow sputum. The doctor thinks it is of viral origin and
decides not to prescribe antibiotics. The doctor also recommends that he sees the pharmacist for some
cough mixture. Which of the following active ingredients would be best in the cough mixture?

A Guaifenesin B. Codeine C. Pholcodine D. Dextromethorpha

354) What is the most important ingredient in maternal multi-vitamin products?

A Folic acid B. iron C. Copper D. Beta-carotene

355) An 89-year-old man with benign prostatic hypertrophy (BPH) has been on alpha blocker for eight
months. On his routine follow up he is complaining of lower urinary tract symptoms of difficulty voiding.
The doctor has decided to start him on a Phosphodiesterase-5 inhibitor in addition to the a-blocker. Which
of the following combination is most appropriate?

A. Tadalafil 4 hours after Tamsulosin


B. Tadalafil at the same time as the Tamsulosin
C. Sildenafil 4 hours after Tamsulosin .
D. Sildenafil at the same time as the Tamsulosin

356) A 79-year-old woman has urinary urgency, frequency and bed wetting at night. She has a history of
osteoarthritis and hyperthyroidism. Which of the following interventions is most appropriate?

Darifenacin A. Pelvic floor exercise C. Tolterodine D. Oxybutynin

357) A 73-year-old woman has a three-week history of weight loss, heat intolerance, palpitations and
tachycardia. On examination she is anxious and has warm extremities. She is not fit for ablative therapy
and the doctor wants to try medications instead (see lab results). What would be the best initial choice of
drug? TSH 0.12 ( 0.4 - 0.5 Tyroxi ( T4) 25 (8.5 - 15.2)

B. Methimazole A. Propranolol C. Levothyroxine D. Ipodate sodium

358) A 23-year-old drug addict man was brought to the Emergency Room with severe agitation and
excitement. He was administered intravenous diazepam, with no improvement in his condition. Which of
the following is the mostly likely drug the patient was addicted on?

A.Opioids B.Cocaine C.Pregabalin. D.Cannabinoids

359) Phenobarbital precipitate with? - Cyclizine Dextrose 10% Water

360) How to prepare Phenobarbital? - Water Dextrose10%

Dr Aamir Iqbal 47 Dr Akhtar Hussain


361) A patient Fasting blood sugar found 3.5millimol/Liter. He is taking insulin what would be the
preferable advice that can be given by the pharmacist

a) Change in to OA D
b) Reduce the dose of Insulin
c) The preferable maintenance of blood glucose level should be between 4-6 millimol/liter

362) A patient is taking long-acting insulin 16U in the morning and ultrashort acting insulin 3U three times
in a day. But the patient HbA1C found to be 7.6% and he showed the every recorded FBS and PPBS which
were also high. what is the most advice can be given to the patients

a) Insulin should be taken after the meal.


b) Insulin dose should be titrated according to the day-byday sugar level.
c) One more dose of Long acting insulin administered in night time
d) The site in which long acting insulin administered should not be used for short acting insulin

363) An 18 year-old boy, weighing 60 kg, is diagnosed with type I diabetes. He is prescribed a basal/bolus
insulin regime of 0.5 U/kg to mimic physiological levels of insulin as closely as possible. Which would be the
most appropriate initial basal insulin regimen?

A. 30 units of insulin Aspart once daily


B. 30 units of insulin Glargine once daily
C. 20 units of insulin Levemir once daily
D. 15 units of insulin Glargine once daily

364) A 65-year-old woman received a diagnosis of RA 1 year ago. At the time of her diagnosis, her RF titer
was 1:64; she presented with joint inflammation in both hands and about 45 minutes of morning stiffness.
She began therapy with oral methotrexate and currently receives 15 mg weekly, folic acid 2 mg daily,
ibuprofen 800 mg three times daily, and omeprazole 20 mg daily. At today’s clinic visit, the patient reports
the recur- rence of her symptoms. Radiographic evaluation of her hand joints shows progression of joint
space narrow- ing and bone erosion. Which is the next best step for this patient’s treatment?

A. Administer etanercept..
B. Change to leflunomide.
C. Add prednisone bridge therapy.
D. Switch to hydroxychloroquine.

365) A 28-year-old woman has been taking fluoxetine for depression and now has worsening depressive
episodes. The resident doctor on the floor has been asked by consultant to stop fluoxetine and initiate
phenelzine. How long should the washout period be before starting phenelzine?

A. 2 weeks B. 3 weeks C. 4 weeks D. 5 weeks

366) What is the pharmacological category for insulin lispro?

A. Long-acting insulin B. Rapid-acting insulin C. Short-acting insulin D. Intermediate-acting

Dr Aamir Iqbal 48 Dr Akhtar Hussain


367) How many grams of 5% diclofenac cream should be mixed with 100 g of 1% cream to make a 2.5%
diclofenac cream?

A. 30 g B. 60 g C. 90 g D. 120 g

368) Which NSAID permanently inactivates thromboxin-A2 synthesis in platelets?

A. Aspirin B. Naproxen C. Celecoxib D. Ibuprofen

369) Which of the following medications can be used to lower the elevated serum potassium
concentration?

Kayexalate Sevelamer Dextrose Sodium bicarbonate

370) What is the appropriate mode of taking chewable aspirin tablet?

A. Should be chewed .
B. Should crushed and dissolved in 30 ml of water. .
C. Should be swallowed .
D. Can be chewed or swallowed.

371) A content uniformity test for tablets is endorsed by the United Stated Pharmacopeia to ensure which
quality?

Potency Bioavailability Stability Solubility

372) What is the bioavailability of a diclofenac intramuscular injection?

Approximately 100% 75% 25% 50%

373) Which one of the following medications can cause weight loss?

A. Orlistat B. Olanzapine C. Atorvastatin D. Glibenclamide

374) A 54-year-old man with reduced ejection fraction heart failure presented to the pharmacy with mild
pain in his knees and requesting an OTC pain killer. Which of the following is the best OTC medication?

Paracetamol Ibuprofen Celecoxib Naproxen

375) Which of the following is a benefit of the formulary system?

Increased safety Decreased cost Physician satisfaction

376) Which of the following parenteral anticoagulants require routine monitoring of coagulation lab
parameters?

UFH intravenously UFH subcutaneously Enoxaparin subcutaneously

Dr Aamir Iqbal 49 Dr Akhtar Hussain


377) Which of the following P2Y12 receptor inhibitors exhibit the most variable anti-platelet effects?

A. Prasugrel B. Ticagrelor C. Ticlopidine D Clopidogrel

378) Stroke or previous TIA is a contraindication to the use of which anti-platelet agent?

Prasugrel Ticagrelor Clopidogrel Aspirin

379) What is the side effect associated with calcium carbonate containing antacids?

Constipation Weight gain Hypercalcemia Increase salivation

380) A 75-year-old man has recently been diagnosed with open angle glaucoma. He is prescribed topical
eye drops once daily. He presents to the pharmacy with the prescription for eye drops labeled, "instill one
drop daily in both eyes". What would be the best time to administer the eye drops?

A. At night B. In the morning C. In the afternoon D. Anytime of the day

381) A 79 year-old woman with a history of open angle glaucoma presents to the pharmacy asking for
advice on how long to wait between instilling her second eye drops. She has been prescribed two different
eye drops and prefers to instill the eye drops when she goes to sleep. How long should she wait to
administer the second eye drops?

5 minutes 3 seconds . 3 minutes 5 seconds

382) Which of the following Which of the following antineoplastic medications is known to cause bone
marrow suppression?

Daunorubicin Vincristine Bleomycin L -asparaginase

383) The high prevalence of osteoporosis in the elderly results which incidence in geriatric population?

Bone fractures Urinary retention Memory impairment

384) Which dosage form is appropriate when maximum moisturization is required?

Ointment Cream Lotion Gel

385) Caretaker visits pharmacy every day to collect medicine to the patient. Which medicine increased the
pharmacist concern?

- Zolpidem - Glucosamine - Some vitamins

386) Which ONE of the following beta-blockers would be LEAST likely to cause bronchospasm in an
asthmatic patient at normal doses?

- Carvedilol - Labetalol Bisoprolol - Propranolol

Dr Aamir Iqbal 50 Dr Akhtar Hussain


387) Gray baby syndrome produced by:

Chloramphenicol Gentamicin benzyl penicillin

388) Who is contraindicated to use amorolfine nail lacquer?

Patient with CHF Patient with Renal disease Patient with diabetes

389) Drug is secreted at Brest fed infant and cause hypnosis.

. Diazepam A. Haloperidol B. Chlorpromazine

390) Which of the following anti-diabetic medications should be used as a first line agent in patients with
pre-diabetes?

Glyburide Metformin Insulin Pioglitazone

391) A 54-year-old man is diagnosed with stage 1 hypertension. His left ventricular ejection fraction is less
than 40%. What would be the most appropriate drug for him?

Lisinopril A. Amlodipine C. Spironolactone D, hydrochlorothiazide

392) A patient is in some sort of pain and he is allergic to diazepam. He needs analgesic. Which of the
following he should take:

Amitriptyline Ibuprofen Naproxen diclofenac

393) Patient with depression and have headache that do go away although he used a lot of analgesics.
What contributes to his headache? b.

Headache is due to misuse of analgesics Depression causes his headache

394) Which of the following counselling points is very important to discuss with patients presenting with
constipation?

Increase water intake throughout the day.


Decrease fiber intake in your diet
Increase fat in your diet
Stay in the bathroom for at least 30 minutes in the morning.

395) What is the most common side effect(s) of niacin that should be mentioned to patients during
counseling?

Chills Dry cough Tinnitus Flushing and dyspepsia

396) preferred rout for peptide/protein administration ?

Oral IV IM SC

Dr Aamir Iqbal 51 Dr Akhtar Hussain


397) Patient is having ulcerative colitis and using mesalazine and corticosteroid. What mineral should she
take:

Calcium VitaminB12 Magnesium Iodine

398) Inpatient with MI taking 5 drugs (prasugrel, bisoprolol, aspirin, & another 4th), upon discharge he
found only 4 drugs what is the missing one?

Amlodipine Warfarin, Ramipril

399) Which of the following medication can be put together in the same ward?

Furosemide and warfarin


Methotrexate and Lisinopril
Dispersible aspirin and atorvastatin

400) What is a high alert drug? Warfarin Doxazocin

401) Which is the anticoagulant preferred during hemodialysis procedure?

A. Unfractionated heparin B. Apixaban C. dabigatran D. Enoxaparin

402) Which one of the following is known side effect of unfractionated heparin?

Hyperkalemia A. Hypokalemia C. Hypocalcemia D. Hypercalcemia

403) What would be the expected change in the clearance of medications in acute renal failure?

Clearance of drugs eliminated mainly by the kidney decreases.


Clearance of drugs eliminated mainly by the kidney increases.
Clearance is a constant parameter and it's affected by the drug dose.
Clearance is a constant parameter and it's affected by the drug concentration.

404) Someone asked about the diet plan & you didn’t know it. What you should do?

Refer him to dietician - Tell him you don’t know - Search, learn and inform him

405) A physician would like to replace patient's valproic acid with immediate release lamotrigine. What is
the recommended maximum initial dose?

A. 200 mg B. 300 mg C. 400 mg D. 500 mg

406) Marketing company reached you to take information about the patient for marketing reasons:

Pharmacist refuses because of unclaimed information


Pharmacist refuses because it's not health promotion goal

407) Which one of the following medications can be dispensed without prescription?
Ibuprofen 400 mg A. Metoprolol 100 mg C. Insulin vials D. Aripiprazole 10 mg

Dr Aamir Iqbal 52 Dr Akhtar Hussain


408) What is the total maximum daily dose of over-the-counter Ibuprofen?

1200 mg 3000 mg 2000 mg 800 mg

409) According to Healthcare authority -Abu Dhabi , which of the following case should the prescribers use
the generic name only in prescription

Ampicillin Insulin preparation OTC Digoxin

410) A 21-year-old woman with a history of chronic migraine associated with aura is asking to for
medication to prevent another attack. What would be the recommended prophylaxis?

Propranolol 20 mg twice daily


Topiramate 25 mg twice daily
Amitriptyline 50 mg at night
Sumatriptan 50 mg daily

411) A drug that causes serious interaction with simvastatin

A. Amiodarone B. Lidocaine C. Quinine D. Ciprofloxacin

412) Amiodarone may increase the serum concentration of Simvastatin Simvastatin interacts with?
Grapefruit

413) Avoid combination Lamicital 100 mg and Lamisil 25 mg are two drugs with similar spelling, how to
arrange them?

A. Expiry B Generic C. Strength D. Brand

414) how many Elemental iron in ferrous sulfate 200 mg?

A. 65 mg B. 75 mg C. 100 mg D. 200 mg

415) What information should be provided to a patient on fentanyl transdermal patches?

A. Avoid increase in body core temperature


B. Rotate the site of application to avoid tolerance.
C. Store patches in the refrigerator to decrease stinging sensation.
D. Apply heating pads on the transdermal patch to improve absorption.

416) An 18-year-old boy with a history of illicit drug use is suspected to have an overdose of
benzodiazepines and is experiencing ataxia, drowsiness and nystagmus. What would be the best antidote?

Flumazenil 300 mcg intravenously.


Activated Charcoal 50 grams
Naloxone 400 mcg intramuscularly
Haloperidol 10 mg intramuscularly

Dr Aamir Iqbal 53 Dr Akhtar Hussain


417) The physician is asking the pharmacist about when to discontinue clopidogrel before open-heart
surgery. What should the pharmacist suggest?

B. Discontinue 5 days before surgery. .


C. Discontinue 3 days before surgery. .
D. Discontinue 2 weeks before surgery. .
E. Risk of bleeding is minimal, continue clopidogrel.

418) A 65-year-old man with a history of chronic NSAID use presents with melena. the endoscopy shows
erosive esophagitis. Which of the following drugs will be most appropriate treatment?

A. Ranitidine B. Sucralfate C. Lansoprazole D. Metoclopramide

419) A 45-year-old man with bipolar disorder has been stable on lithium for 12 months. He is due for his
routine laboratory work up. Which of the following electrolyte imbalances will predispose him to lithium
toxicity?

A. Mg B. K C. Ca D. Na

420) Doctor wrote Dalacin T (DNS) in the prescription. What is the meaning?

Do not substitute A. Do not dispense in brand name B. Do not dispense in generic

421) A patient who is on warfarin therapy takes ginger supplement, to see any interactions, which
reference would be most appropriate?

A. Red book B. Merk manual C Review of natural products D. Handbook on injectable drugs

422) Which of the following vaccine is CI in pregnancy?

Varicella zoster vaccine(another example MMR)


Hepatitis B vaccine
Meningococcal polysaccharide vaccine
Influenza virus vaccine

423) What is the content of monophasic contraceptive pill?

Constant dose of estrogen and progesterone


A. Constant dose of estrogen .
B. Variable dose of progesterone .
C. Variable dose of estrogen and progesterone

424) Which of the following replacement solution is isotonic?

Ringer’s lactate A. 3% saline C. Dextrose 2.5% D. 9% saline

Dr Aamir Iqbal 54 Dr Akhtar Hussain


425) Where to prepare cytotoxic medicine?

A. Horizontal laminar flow hood


B. Reverse laminar air flow hood
C. Fume hood
D. Vertical laminar flow hood

426) Rice water stool is a symptom of ? A. Dengue B. Malaria C. Cholera D. Typhoid

427) Drug to be wrote in brand name. - Tegretol (carbamazepine) - Lyrica - Tritace - Lamictal

428) Which is the first line agent to treat glaucoma in pregnancy?'

A. Beta-blockers B. Prostaglandin analogue C. Brimonidine D. Pilocarpine

429) Wrong statement regarding narcotic prescription:

A. New prescription must be written every time


B. Schedule II medications can be refilled
C. Medications classified as a schedule III and IV controlled substances may be refilled .
D. All of the above

430) When given ethambutol for a long duration, continuous follow up/ checkup is necessary because of
this condition:

A. Renal failure B. Liver failure C. Optic neuritis D. Myopathy

431) A 25-year-old woman on Levothyroxine become pregnant. She now complains of persistent fatigue.
What should be done?

A. Increase dietary Iodine.


B. Double the dose of levothyroxine.
C. Do nothing, fatigue is normal in pregnancy.
D. Adjust thyroxine dose based on TSH value during the first trimester.

432) Corticosteroids given in sepsis for?

Decrease staying in ICU A. Mortality B. Decrease pain scale D. Decrease needs for fluids

433) Why nitrofurantoin cannot be given with abnormal patient GFR? -

GFR is low it will precipitate in liver no need to antibiotic - GFR is too high it will excrete

434) A 31-year-old, three months pregnant woman, develops urinary tract infection. What is the most
appropriate antimicrobial therapy?

Nitrofurantoin B. Ciprofloxacin A. Tetracycline D. SMX/TMP

435) Drugs with pka =5 absorption site? – Duodenum

Dr Aamir Iqbal 55 Dr Akhtar Hussain


436) At which pH aspirin (pka 3.5) will be most soluble? 6 5 3 2

437) Empty narcotic drug ampoule should be?

Discard in Sharp container Save in narcotic drug cabinet Put in chemical bin

438) What does "scored tablet in quarter" indicate?

Tablet can be broken in half or quarters Tablet can be broken in half

439) A 55-year-old man with a BMI of 37 is diagnosed with Type II diabetes three months ago. He failed to
adhere to prescribed diet and exercise regimen. Which of the following medications would be best
indicated?

A. Metformin B. Gliclazide C. Tolbutamide D. Glibenclamide

440) Which route of administration results in faster absorption?

A. Oral B. Rectal C. Topical D. Subcutaneous

441) An eight-month-old boy with recurrent otitis media has been taking high dose amoxicillin (90 mg/kg)
for seven days. There is no fever and no evidence of hearing loss. He is due his 3rd dose of pneumococcal
vaccination and the influenza vaccination. The nurse is asking about due vaccination. What would be the
best action?

Proceed with pneumococcal and influenza vaccination.


Cancel pneumococcal and influenza vaccination.
Postpone pneumococcal and influenza vaccination for one year.

442) Which one of the following is considered first line therapy for the management of partial seizures?

A. Carbamazepine B. Phenobarbital C. Gabapentin D. Primidone

443) What is the appropriate treatment of poisoning by organophosphate nerve agents?

A. Pralidoxime B. Hemodialysis C. Gastric lavage D. Activated Charcoal

444) Patients take phenytoin and take some steroids which drug causes vomiting and irregular heart rate
and confusion and dizziness Cause??

A digoxin B. Furosemide C. Simvastatin

445) A 21-year-old man with a history of illicit drug use is suspected to have overdose of an opioid and is
experiencing severe toxicity with respiratory depression and pin-point pupils. What would be the best
antidote?

Naloxone 400 mcg intramuscularly


Activated Charcoal 50 grams.
Diazepam 10 mg intramuscularly
Flumazenil 300 mcg intravenously
Dr Aamir Iqbal 56 Dr Akhtar Hussain
446) Which of the following medications can be teratogenic if a pregnant women handles crushes or
breaks the tablet?

A. Bosentan B. Alfuzosin C. Topiramate D. Didanosine

447) Which one of the following medications for osteoporosis will significantly reduce the risk of hip
fracture?

. Risedronate A. Ibandronate C. Raloxifene D. Calcitonin

448) Which one of the following is a property of well-formulated suspension?

A. Re-suspend upon moderate shaking


B. Require small amount of preservative
C. Form cake at the bottom of the bottle
D. Separate the powder from the solution easily upon shaking

449) Which one of the following medications is considered a high alert drug?

A. Insulin B. Finasteride C. Ceftriaxone D. Ciprofloxacin

450) Which of the following laxatives have the slowest onset of action?

A. Psyllium B. Glycerin C. Bisacodyl D. Mike of Magnesia

451) How do the proton pump inhibitors exert their pharmacological action?

Inhibiting H/K-adenosine triphosphate in gastric parietal cells


Increases gastrointestinal motility
Inhibits epithelial growth factor in the stomach
Stimulates histamine-2 receptors in the gastric parietal cells

452) A 58-year-old man, recently diagnosed with stage-I hypertension, was started on a low dose ACEI a
week ago. The aim is to titrate the dose to target dose within four weeks. On routine follow-up, he is
complaining of persistent cough. What would be the best action?

Substitute the ACEI with a ARBs


Substitute the ACEI with a calcium channel blocker
Stop the ACEI
Continue titrating the ACEI and ignore the cough

453) A 24-year-old man is diagnosed with psychosis. He agrees to start drug therapy, however, he is asking
for the least sedating medication. Which of the following drugs will be most appropriate?

A. Clozapine B. Quetiapine C. Olanzapine D. Risperidone

454) Which of the following drugs would have contributed to the abnormalities in lab results
Megnesium 0.4 ( 0.7-1.2 )?
A. Omeprazole B. Cimetidine C. Ranitidine D. Gaviscon

Dr Aamir Iqbal 57 Dr Akhtar Hussain


455) A 28-year-old expectant mother who is diagnosed with Group B hemolytic Streptococcus and has
received three doses of ampicillin during labor, delivery is through natural means. Routine examinations of
baby show high grade fever, jaundice, and signs of respiratory distress. What would be the optimal empiric
antibiotic treatment?

Ampicillin and Gentamicin


Not required
Ampicillin and Ceftriaxone
Ceftazidime and Gentamicin

456) Which of the following patient categories the gentamicin extended interval regimens (once daily
dosing) is most suitable?

Gram-negative infections Pregnancy Burns patient

457) Patient taking gentamicin, which of the following are increased side effects:

Increased thirst must be report to doctor

458) What is the total maximum daily dose of the over-the-counter paracetamol?

4g 2g 6g 8g

459) What is the best time to do lithium levels after the dose is administered?

A. 4 hours B. 6 hours C. 8 hours D 12 hours

460) A 48-year-old man is on phenytoin 100 mg TID for epilepsy. His dose is adjusted to 150 mg TID. When
will it be appropriate to do plasma phenytoin concentration at the new steady state?

7 days 30 days 3 days 1 day

461) A 79-year-old woman has urinary urgency, frequency and bed wetting at night. She has a history of
osteoarthritis and hyperthyroidism. Which of the following interventions is most appropriate?

A. Pelvic floor exercise B. Darifenacin C. Tolterodine D. Oxybutynin

462) A 60-year-old man with chronic kidney disease for five years, hypertension, coronary artery disease
and diabetes mellitus is due for routine follow up (see lab results (Phasphate 2.5 (0.8-1.5). Which of the
following drugs is most appropriate?
A. Sevelamer B. Calcium C. Calcium Carbonate D. Aluminium Carbonate

463) Patient with hypertension was rushed to the ER due to signs and symptoms of acute kidney injury.
These are the drugs she/he were taking: Candesartan, Atorvastatin, Aspirin, Ibuprofen, Bisoprolol,
Paracetamol. What drugs should be discontinued?
- Stop candesartan, aspirin and ibuprofen
- Stop candesartan, bisoprolol and atorvastatin
- Stop candesartan, aspirin and paracetamol
- Stop candesartan, atorvastatin and paracetamol

Dr Aamir Iqbal 58 Dr Akhtar Hussain


464) What is the antidote of acetaminophen?

A. Ethanol B. Naloxone C. Methylene Blue D. N-Acetylcysteine

465) A 60-year-old man with a history of angina pectoris is complaining of shoulder pain. Which of the
following analgesics inversely affect his condition?

A. Acetaminophen B. Celecoxib( Meloxicam ) C. Morphine D. Codeine

466) Patient is taking morphine tablets 60mg every 12 hours and a breakthrough dose when needed..
Doctor switched the patient to Fentanyl patch.. What would be the dose:

a) 120mcg over 12 hours


b) 60mcg over 12 hours
c) 90mcg over12hours
d) 30 mcg every 12 hours

467) which of the following not used in treatment of acute pain.?

Morphine fentanyl naloxone dexamethasone

468) An 88-year-old frail woman with osteoarthritis has difficulty in walking and pain. She has no other
medical history and has failed numerous aids to help her with the pain. Which of the following drugs is the
best recommendation?

A. Acetaminophen B. Glucosamine D C. Celecoxib D. Ibuprofen

469) Which one side effects is known to bisphosphonates?

Osteonecrosis of the jaw Hypotension Muscle pain Headache

470) A 28-year-old woman with seizure is planning to start a family. She is taking folic acid 400 mcg per day.
She is asking about safe epilepsy medications during pregnancy. Which of the following would be the best
medication?

A. Carbamazepine B. Valproic acid C. Lamotrigine D. Phenytoin

471) Which of the following groups is with high physical dependence? Benzodiazepines

472) A diabetic patient on antipsychotic (Olanzapine) , and the doctor wants to change it to another
antipsychotic, what is the best alternative?

Clozapine Haloperidol Quetiapine Aripiprazole

473) A Patient has high level of free drug level of phenytoin because of what? Hypoalbuminemia

Dr Aamir Iqbal 59 Dr Akhtar Hussain


474) The physician is asking pharmacist about the effects of low albumin and acute renal failure on
phenytoin serum concentration. The patient is receiving phenytoin 300 mg once daily. What should be the
most appropriate suggestion?

A. In hypoalbuminemia and acute renal failure obtain free phenytoin serum concentration. .
B. In hypoalbuminemia and acute renal failure obtain peak phenytoin serum concentration. .
C. In hypoalbuminemia and acute renal failure obtain total phenytoin serum concentration. .
D. In hypoalbuminemia and acute renal failure obtain trough phenytoin serum concentration

475) patient with serum phenytoin level is 11 mg/dl ( normal 10-20 ) what is your intervention?

A- no need to dose adjustments B- increase dose C- decrease dose

476) New nurse is giving adolescent injection in gluteus muscle and doesn’t know what is maximum
volume of injection :

5ml 1ml 3ml 10ml

477) Most suitable form of infants : Syrup Tablet

478) What is the recommended hemoglobin target when treating anemia due to chronic kidney disease
with epoetin Alfa?

A. 11 g/dL B. 13 g/dL C. 14 g/dL D. 16 g/dL

479) In which condition nasal spray of Steroid is contraindicated

a) Rhinitis b) Nasal infection c) Post nasal surgery d) Deviated septum

480) A 15-year-old boy or 15 month is brought to the Emergency Department by his parents with high
grade fever, vomiting, and non-balancing rash. He started immediately on cefotaxime. The cerebral spinal
fluid culture reveals Neisseria meningitis. What would be the recommendation regarding prophylaxis of his
family and close contacts?

A. His parents should receive rifampicin


B. Antibiotic prophylaxis is not indicated
C. Patient's close contacts should receive rifampicin

481) 73 old man with uti and penicillin allergy what may be the drug given?

A. Amoxicillin B. Trimethoprim C. Ceflaxin D. Nitrofurantion

482) Which of the following antibacterial drug remain longer duration of action?

A. Benzathine penicillin B. Ethambutol C. Azithromycin D. Kanamycin

Dr Aamir Iqbal 60 Dr Akhtar Hussain


483) A 55-year-old woman presents with recurrent symptoms of H. pylori infections. She was previously
treated with the classical amoxicillin, clarithromycin, omeprazole regimen. The clinic is unable to obtain
susceptibility testing for the Helicobacter pylori but the pattern of local antimicrobial resistance is well
known. What would be the optimal treatment for her?

A) Bismuth subsalicylate (2) and Tetracycline (500 mg) QID plus Metronidazole (500 mg) and Omeprazole
(40 mg) BID for 14 days
B) Amoxicillin (1 g), Clarithromycin (500 mg) and Metronidazole (500 mg) plus Omeprazole (20 mg) BID for
14 days
C) Amoxicillin (1 g), Clarithromycin (500 mg) and Metronidazole (500 mg) plus Omeprazole (40 mg) BID for
14 days
D)Bismuth subsalicylate (2) and Doxycycline (100 mg) QID plus Metronidazole (500 mg) TID and
Omeprazole (40 mg) BID for 14 days

484) Patient took a drug, no harm happened but requires additional monitoring what category of error? -

Category A - Category B - Category C - Category D

485) An ophthalmic drop of drug X has 0.1% concentration. Express this concentration of drug X in parts
per million?

A. 10 ppm B. 100 ppm C. 1000 ppm D. 10000 ppm

486) Which of the following of anti-anginal agents reduce myocardial ischemia by dilating coronary arteries?

A. Atenolol B. Amlodipine C. Hydrochlorothiazide D. Isosorbide dinitrate

487) In patients who require dual antiplatelet therapy, what is the recommended maintenance dose of
aspirin when used in combination with ticagrelor?

A. 325 mg B. 162 mg C. 81 mg D. 50 mg
The use of aspirin above 100 mg decreases effectiveness of ticagrelor

488) What is the relationship between childhood vaccines and autism?

A. MMR vaccine causes autism


B. Any vaccine can cause autism
C. Hepatitis B vaccine causes autism
D. None of the vaccines causes autism

489) What is the suitable dosage form for infants?

A. Oral syrup B. Capsule C. Tablet D. Caplet

490) What is the monitoring parameter for warfarin therapy?

A. CT B. INR C. aPTT D. PT:aPTT Ratio

Dr Aamir Iqbal 61 Dr Akhtar Hussain


491) Treatment of malignant hypertension :

a) methyldopa b) propranolol c) nitroprusside sodium d) digoxin

492) Status epilepticus treatment : a) Gabapentine b) diazepam

493) Infant has low graded fever what to do:

a) give ampicillin 5ml every 8 hours


b) give paracetamol drop 20mg/kg
c) give ibuprofen 15mg/kg
d) refer to emergency

494) One of these vaccines can be recommended to a pregnant

a) measles, mumps and rabies vaccine


b) rota virus vaccine
c) varicella zoster vaccine
d) meningococcal vaccine

495) An error occurred but the error didn’t reach the patient. This error can be categorized as.

A. Category A B. Category B C. Category I D. Category N

496) What is the effect of thiazide dietetics on serum lithium concentration?

A. Concentration will increase .


B. Concentration will decrease .
C. Minor variable changes.
D. No changes

497) About Granulocyte colony stimulating factor (G-Csf) administration?

- 24-72 hrs. after chemotherapy

498) 5-flurouracil given to treat patient 15mg/kg and is diluted in 500ml . patient is 61 kg and 5- flurouracil
is given as 1g in 20 ml. calculate the amount of the drug?.

Calculate the total dose required:


The patient weighs 61 kg.
The prescribed dose is 15 mg/kg.
Total dose = 61 kg * 15 mg/kg = 915 mg.
Determine the volume of 5-flurouracil solution needed:
The concentration of 5-flurouracil is 1g in 20 ml, which is equivalent to 1000 mg in 20 ml.
To find out how many ml are needed for 915 mg,
we set up a proportion: (1000 mg / 20 ml) = (915 mg / X ml).
Solving for X, we get X = (915 mg * 20 ml) / 1000 mg = 18.3 ml.

Dr Aamir Iqbal 62 Dr Akhtar Hussain


499) It was about TPN 5,000 ml and the rate of administration And there was 10% overage My confusion
was whether the overage was included in the 5 L or not or
Patient prescribed TPN 5,000 mL, including overage, daily. The overage was 10% and it was to be giving
over 12 HR. Calculate the rate to administer: My answer was 6.~ ml./min

Given the information, the patient is prescribed TPN 5,000 mL daily,


and it's specified that this volume includes a 10% overage.
This means the original prescribed volume before adding the overage is less than 5,000 mL, and the
overage has been added to reach this total volume.
To find the original prescribed volume (OPV) before overage, we can set up the equation where OPV plus
10% of OPV equals 5,000 mL.
Mathematically, this is represented as OPV + (0.10 * OPV) = 5,000 mL, simplifying to 1.10 * OPV = 5,000 mL.
Solving for OPV gives us OPV = 5,000 mL / 1.10 = 4,545.45 mL.
However, since the total volume to be administered, including the overage, is 5,000 mL over 12 hours, we
calculate the rate as follows:
Total volume (5,000 mL) divided by total time in minutes (12 hours * 60 minutes/hour = 720 minutes).
Systematically Ensure Accuracy & Precision
Re-evaluating our calculation to ensure accuracy, we confirm that the total volume of 5,000 mL is to be
administered over 12 hours. Dividing this volume by the total administration time in minutes (720 minutes)
gives us the correct rate of administration.
Final Answer
The rate to administer the TPN, including the 10% overage, over 12 hours is 5,000 mL / 720 minutes = 6.94
mL/minute.

500) 0.2 to 0.4 mg/ml max concentration. Make etoposide in 250ml NS. Vial volume around 100mg/5ml .
Choose the volume of concentrated etoposide to add in NS

A: 10 ml B: 5 ml C: 8 ml

The correct answer is:


Here's how to calculate the volume of concentrated etoposide needed:
1. Desired concentration range: 0.2 mg/ml to 0.4 mg/ml
2. Stock concentration: We need to know the concentration of the etoposide in the vial. it's around
100mg/5ml. This translates to a concentration of 100mg / 5ml = 20 mg/ml.
3. Volume of normal saline (NS): 250 ml
Formula:
Volume of etoposide = (Desired concentration * Volume of NS) / Stock concentration
Calculation:
We want to achieve a concentration within the desired range.
Let's calculate for the lower limit (0.2 mg/ml):
Volume of etoposide = (0.2 mg/ml * 250 ml) / 20 mg/ml = 2.5 ml
Since we want to ensure the final concentration falls within the entire desired range (up to 0.4 mg/ml), we
need to add a little more etoposide. Adding *5.5 ml* would be enough:
Additional volume = 0.4 mg/ml - 0.2 mg/ml = 0.2 mg/ml
Additional volume of etoposide = (0.2 mg/ml * 250 ml) / 20 mg/ml = 2.5 ml
Therefore, adding 2.5 ml from the previous calculation and 2.5 ml for the additional volume gives us a total
of 5 ml.

Dr Aamir Iqbal 63 Dr Akhtar Hussain


501) The patient was taking allopurinol and he had rash. He came to refill or take medication again, what
you will do? - Don’t refill, don’t give

Allopurinol may cause a severe, life-threatening skin rash. Patients should be advised to cease allopurinol
and seek medical attention if rash or itch appears.

502) Patient taking some drugs like; Aluminum hydroxide, Magnesium silicate, Beta blocker…, now
prescribed doxycycline. What is the advice?

- Administer Doxycycline with meals (to avoid GI SE) & 2 hrs. after or before aluminum hydroxide &
magnesium silicate administration (because it decreases their absorption)

503) The patient is taking warfarin and the doctor prescribed fluconazole. The patient came with INR=11
and he’s bleeding. What is the reason of this bleeding? (normal INR 1.1 or less if warfarin 2-3 )

Fluconazole is an enzyme inhibitor & causes warfarin to accumulate in the blood without metabolizing it.

504) Side effect of trimethoprim e)

Nephrotoxicity Ototoxicity Hepatotoxicity

505) Off label use of cetirizine what to do? Or When doctor write off label use for one medicine what to do?

- Inform patient about the use


- Not your responsibility, doctor must inform him
- Responsibility of manufacturer

506) DOAC (Direct Oral Anticoagulant) Apixaban/rivaroxaban/dabigatran What is not a concern of testing
or least monitored? Activated partial thromboplastin time aPPT In patients on DOAC therapy,

clinicians cannot rely on normal activated partial thromboplastin time (APTT) and prothrombin time (PT)
results to reflect the patient’s level of anticoagulation.

507) Lamicital 100 mg and Lamisil 25 mg are two drugs with similar spelling, how to arrange them?

A. Expiry B. Generic C. Strength D. Brand

508) Piperacillin 4g, tazobactam 0.5g ordered by a nurse to be given to a patient with a history of breast
cancer. She is febrile 39C, 110/80, and tired. (not mentioned if she has drug allergy).

Pharmacists refuse to give, order serum levels first.


Pharmacists refuse to give, order creatinine levels first.
Tell them to order a narrow spectrum antibiotic like flucloxacillin.
Give Piperacillin/Tazobactam until lab report is up.

509) Not a medicine optimization benefit or the least concern?

- Cost effectiveness - Can be added to routine Safety

Dr Aamir Iqbal 64 Dr Akhtar Hussain


Error Categories

Dr Aamir Iqbal 65 Dr Akhtar Hussain


Dr Aamir Iqbal 66 Dr Akhtar Hussain
Dr Aamir Iqbal 67 Dr Akhtar Hussain

You might also like